R REVIEW CARDIO- CONDUCTION D/O

¡Supera tus tareas y exámenes ahora con Quizwiz!

A 67-year-old woman with sick sinus syndrome complains of significant lightheadedness. If she is driving and a "dizzy spell" begins, she has to pull off the road and wait for it to pass. You capture an ECG during one of her spells in the office and note a bradycardia in the 50s. Which of the following is the best treatment recommendation? A Atenolol B Defibrillator C Digitalis D Pacemaker

Correct Answer ( D ) Explanation: Sick sinus syndrome occurs as a result of disease of the sinoatrial (SA) node. It is associated with tachycardia-bradycardia syndrome in which the sinus rate varies from fast to slow and back again. ECG shows an irregular rhythm with pauses in sinus activity. Many patients with sick sinus syndrome respond favorably to permanent pacemaker placement. In fact, the treatment of choice for any symptomatic bradycardia that accompanies sick sinus syndrome is pacemaker placement.

Rapid Review Ventricular Tachycardia

>3 consecutive ectopic ventricular beats Monomorphic, polymorphic Bidirectional: digoxin toxicity Wide complexes Pulseless: immediate defibrillation Unstable: synchronized cardioversion Stable: procainamide, amiodarone, synchronized cardioversion (refractory) If unsure, manage all wide complex tachycardias as ventricular tachycardia

A 26-year-old woman presents with dizziness and palpitations. She reports episodes of these symptoms beginning about 1 week ago, which initially only lasted a few minutes. However, for the past two days, she has had about 4 episodes a day which last about 20 minutes each. Her social history is significant for heavy caffeine intake. Her pulse is 166 bpm and her blood pressure is 140/70. Her rhythm strip is seen above. Which of the following best describes the most likely underlying cardiac pathology responsible for this patient's symptoms? A. Conduction reentry B. Dilated cardiomyopathy C. Myocardial infarction D. Valvular abnormality

A. Conduction reentry Paroxysmal supraventricular tachycardia (PSVT) is an acute onset tachydysrhythmia that can affect any age group. The dysrhythmia is caused by a reentrant conduction pathway, most commonly in the atrium or atrioventricular node. Common triggers include atrial or atrioventricular premature beats, hyperthyroidism, caffeine and drugs. The common symptoms of paroxysmal supraventricular tachycardia include palpitations, dizziness, dyspnea, syncope, angina, fatigue, nausea and sweating. The most common symptoms are palpitations and dizziness. The chest pain which accompanies this class of dysrhythmias is usually due to the tachycardia itself. Hemodynamically stable patients should first attempt vagal maneuvers. If unsuccessful, AV-nodal blocking agents can be administered such as adenosine or calcium channel blockers. If the patient is hemodynamically unstable, she should immediately undergo cardioversion. One Step Further Question: What is the long-term treatment for paroxysmal supraventricular tachycardia (PSVT)? Answer: Calcium channel blockers, beta-blockers and radiofrequency catheter ablation.

Rapid Review Atrial Fibrillation

Alcohol Irregularly irregular No P waves Narrow QRS unless conduction block or accessory pathway Unstable: cardioversion Stable: Rate control with CCBs, ßBs <48 hours duration: cardiovert to sinus rhythm >48 hours duration: anticoagulate, echo to r/o thrombus, then cardioversion

Rapid Review Supraventricular Tachycardia

All tachydysrhythmias that arise above the bifurcation of the bundle of His Characteristics: Atrial rate 120-200 Rhythm: regular Narrow QRS Causes Pre-excitation syndromes (WPW) Mitral disease Digitalis toxicity Drugs and toxins Hyperthyroidism Treatment: Vagal maneuvers (Valsalva) Adenosine (first-line medication), ßBs, CCBs Unstable: synchronized cardioversion

One Step Further Question: What is the normal range for the PR interval?

Answer: 120-200 milliseconds.

Which of the following antiarrhythmic medications is contraindicated in the setting of coronary artery or structural heart disease? A Amiodarone B Dofetilide C Dronedarone D Flecainide

Correct Answer Correct Answer ( D ) Explanation: Flecainide is contraindicated in the setting of coronary artery or structural heart disease because of the increased risk of polymorphic ventricular tachycardia. Flecainide is a class IC antiarrhythmic and is an effective agent against both ventricular and supraventricular dysrhythmias. It is often used for atrial fibrillation and its use is sometimes referred to as the "pill-in-the-pocket" approach. In this approach, the patient takes the medication with episodes of paroxysmal atrial fibrillation, rather than taking medication on a daily basis. Flecainide is an attractive agent because of the relatively good side effect profile, efficacy, and ease of use. However, its use is limited, particularly its proarrhythmic effects. Flecainide should also not be used in the setting of sinus or AV node dysfunction, bundle branch block, or long QT syndrome. Patients should take a short acting beta blocker or calcium channel blocker 30 minutes before taking the antiarrhythmic agent or use background rate control therapy. Because of concern of a post conversion pause, patients should be monitored the first time flecainide is used. Dofetilide (B) and amiodarone (A) are class III antiarrhythmics. Class III agents are used for atrial and ventricular arrhythmias but can lengthen the QT interval; for this reason, they are often initiated in an inpatient setting over 3 days to monitor for torsades de pointes. Amiodarone can prolong the QT interval but has not been shown to cause torsades de pointes and does not have to be initiated as an inpatient. Dronedarone (C), the newest agent, can reduce the incidence of hospitalization for cardiovascular events or death in patients with atrial fibrillation or flutter. Dronedarone should not be used in patients with NYHA functional class II or III heart failure with recent decompensation or with class IV heart failure.

Which of the following is the treatment of choice in antidromic atrioventricular reciprocating tachycardia in a hemodynamically stable patient? A Adenosine B Diltiazem C Electrical cardioversion D Procainamide

Correct Answer Correct Answer ( D ) Explanation: Wolff-Parkinson-White (WPW) syndrome is a ventricular preexcitation syndrome syndrome that occurs due to an anatomic accessory pathway that bypasses the AV node and forms a direct electrical connection between the atria and ventricles, increasing the risk for supraventricular tachydysrhythmias. The most common tachydysrhythmia seen in WPW syndrome is atrioventricular reentrant (or reciprocating) tachycardia (AVRT), which can either be antidromic or orthodromic. Antidromic AVRT accounts for approximately 10% of symptomatic tachydysrhythmias seen in WPW patients and occurs due to anterograde conduction of the atrial impulse through the accessory pathway with return of the impulse through the AV node. The rate is typically very rapid due to the ability of the accessory pathway to conduct the impulse faster than the AV node. The ECG typically shows a rapid wide-complex tachycardia that resembles ventricular tachycardia. This is in contrast to orthodromic AVRT, which is typically a narrow-complex tachycardia. Procainamide is the treatment of choice in hemodynamically stable wide-complex tachydysrhythmias in WPW syndrome. Electrical cardioversion can be used if procainamide fails or in patients with hemodynamic instability. Drugs that slow conduction through the AV node, such as adenosine, calcium channel blockers, and beta blockers, should be avoided as they may enhance conduction through the accessory tract, leading to ventricular fibrillation. Definitive treatment of WPW syndrome is ablation of the accessory pathway. Adenosine (A) is the first line pharmacologic treatment of orthodromic AVRT in patients in whom vagal maneuvers have failed. Adenosine, like other AV nodal blockers such as diltiazem (B), should be avoided in antidromic AVRT because its AV nodal blocking effects may enhance conduction through the accessory tract leading to ventricular fibrillation. Electrical cardioversion (C) is the treatment of choice in hemodynamically unstable tachydysrhythmias or in stable tachydysrhythmias in which pharmacologic interventions have failed.

A 72-year-old man has been more fatigued lately. As a result, his primary practitioner orders an electrocardiogram. A P wave is present for every QRS complex and the PR interval is 225 milliseconds in length. Which of the following is the diagnosis? A First degree atrioventricular block B High-grade atrioventricular block C Mobitz type I atrioventricular block D Third degree atrioventricular block

Correct Answer ( A ) Explanation: A first degree atrioventricular (AV) block is characterized by a long PR interval (>200 milliseconds). The normal PR interval is 120 to 200 milliseconds in length. The PR interval represents the time between atrial depolarization (P wave) and the ventricular depolarization (QRS complex). A prolonged PR interval is a sign of a delayed conduction at the AV node before ventricular depolarization. Causes of first degree AV block include a benign increase in vagal tone, drugs that slow AV conduction (e.g., beta-blockers, digoxin), previous myocardial infarction, dilated cardiomyopathy, and some types of muscular dystrophy. Typically, patients exhibit no symptoms and there are no signs on physical exam.

Which of the following vital signs is considered abnormal in a 1-year-old patient? Heart rate of 160 beats per minute Oxygen saturation of 98% on room air Respiratory rate of 26 breaths per minute Systolic blood pressure of 85 mm Hg

Correct Answer ( A ) Explanation: A heart rate of 160 beats per minute would be considered tachycardia in a 1-year-old patient. The average heart rate for a 1-year-old is 120 beats per minute. An oxygen saturation of 98% on room air (B) is within normal limits for all aged patients, including a 1-year-old. The average respiratory rate for a 1-year-old is 26 breaths per minute (C) and is therefore considered normal. The average systolic blood pressure for a 1-year-old is 90 mm Hg (D) with a minimum systolic blood pressure of 72 mm Hg. One Step Further Question: What formula is used to calculate the minimum systolic blood pressure for a child age 1-10 years? Answer: Minimum SBP = 70 + (2 x age in years). Rapid Review Normal Pediatric Heart Rates <1: 100-160 bpm 1-2: 90-150 bpm 2-5: 80-140 bpm 6-12: 70-120 bpm >12: 60-100 bpm

In which of the following clinical scenarios is an implantable cardioverter-defibrillator indicated for the prevention of ventricular dysrhythmias and sudden cardiac death? A A patient with a left ventricular ejection fraction < 35% and heart failure NYHA Functional Class II or III B A patient with a normal left ventricular ejection fraction and asymptomatic structural heart disease C A patient with sustained ventricular tachycardia in the setting of an acute myocardial infarction D A patient with sustained ventricular tachycardia in the setting of hyperkalemia

Correct Answer ( A ) Explanation: A patient with a left ventricular ejection fraction ≤ 35% and heart failure NYHA Functional Class II or III is a clinical scenario in which an implantable cardioverter-defibrillator is indicated. An implantable cardioverter-defibrillator is a small device combining a cardioverter and defibrillator into one implantable unit that is surgically placed in the chest or abdomen. It is battery powered and programmed to detect dysrhythmias, mainly sustained ventricular tachycardia and ventricular fibrillation, which can lead to sudden cardiac death. It has a very high success rate in rapidly terminating ventricular dysrhythmias and evidence shows that it improves survival. Implantable cardioverter-defibrillator implantation is generally considered the first-line treatment for the secondary prevention of sudden cardiac death in patients who have survived an event and for primary prevention in certain high risk populations. Published guidelines exclude cases that are considered "reversible causes." Some of the major indications are as follows. A patient with asymptomatic structural heart disease (B) is not an indication for implantable cardioverter-defibrillator. Only when a patient with structural heart disease, such as hypertrophic cardiomyopathy, has symptoms or unexplained syncope with inducible dysrhythmia on electrophysiological studies is it indicated for an implantable cardioverter-defibrillator. Patients with sustained ventricular tachycardia in the setting of hyperkalemia (D) or acute myocardial infarction (C) are not indicated for an implantable cardioverter-defibrillator. Hyperkalemia and acute ischemia are considered "reversible causes" and correction of the metabolic derangement and revascularization are often adequate measures to reduce the risk of sudden cardiac death.

A 24-year-old woman presents to the Emergency Department with complaints of palpitations and lightheadedness for the past 30 minutes. A rhythm strip is performed and is shown above. She has no significant past medical history and denies any drug or alcohol use. Vital signs show a blood pressure of 132/86 mm Hg and an oxygen saturation of 96 percent on room air. Which of the following medications is the most appropriate for the treatment of this condition? A Adenosine B Amiodarone C Amrinone D Atropine

Correct Answer ( A ) Explanation: Adenosine is the first-line drug indicated for stable narrow-complex supraventricular tachycardia. This drug is effective in terminating rhythms that are due to a reentrant mechanism involving either the sinoatrial (SA) or atrioventricular (AV) node. It is a nucleoside whose half-life is less than 10 seconds, therefore administration must be done through a rapid IV push followed by a saline flush. The mechanism of action involves inhibition of calcium current, which causes hyperpolarization and suppression of calcium-dependent action potentials. This inhibits AV nodal conduction and increases the AV nodal refractory period breaking an arrhythmia that is generated by a re-entrant mechanism. Amiodarone (B) prolongs action potentials by blocking potassium channels and inactivated sodium channels. It is used in a wide variety of arrhythmias, including shock-refractory ventricular fibrillation (VF) and pulseless ventricular tachycardia (VT). It is also used in wide-complex tachycardia of unknown origin and polymorphic VT. Amrinone (C) is not indicated in SVT. It is used in severe congestive heart failure that is refractory to all other agents. Atropine (D) is a drug that used to be indicated for symptomatic sinus bradycardia because the SA node is very sensitive to muscarinic receptor blockade causing an elevation in heart rate.

Which of the following ECG findings is most characteristic of a premature junctional contraction? A Inverted P' wave following the QRS B Peaked T wave C Premature P' D Widened QRS > 120 msec

Correct Answer ( A ) Explanation: An inverted P' (P-prime) wave following the QRS is most characteristic of a premature junctional contraction. A premature junctional contraction is the result of an irritable automaticity focus in the AV junction which fires a premature stimulus that is conducted to, and depolarizes, the ventricles and often the atria as well. The premature junctional contraction captures the atria in a retrograde fashion and the ventricles in an antegrade fashion. Since atrial and ventricular depolarizations are moving in opposite directions from the junctional focus, the premature P' wave is inverted or opposite the QRS. Bottom up depolarizations record as an inverted P' wave in ECG leads with an upright QRS. The retrograde P' wave may appear before, during, or after the QRS complex. A premature P' wave (C) is seen with premature atrial contractions which record as a P' on ECG. An atrial focus is the origin of this premature atrial contraction, not the SA node, so the stimulus produces a premature and unusually shaped P' wave that looks different than the normal sinus generated P wave. A widened QRS > 120 msecs (D) can be induced by intrinsic or extrinsic factors, including but not limited to, a premature ventricular beat, or pre-excitation of the ventricles via a bypass tract as in Wolff-Parkinson-White and intraventricular conduction delays. Intraventricular conduction delays refer to abnormalities in the intraventricular propagation of supraventricular impulses. These abnormalities of propagation give rise to changes in the shape and duration of the QRS complex. Examples include left and right bundle branch block patterns and their variants. Peaked T waves (B) can be seen with hyperkalemia and are not associated with premature junctional contractions.

A 74-year-old man is having a preoperative ECG performed. What is your interpretation of his ECG? AAtrial fibrillation BAtrial flutter CNormal sinus rhythm DSinus tachycardia

Correct Answer ( A ) Explanation: Atrial fibrillation is an irregularly irregular rhythm due to uncoordinated atrial activation and random occurrence of ventricular depolarization. The atria are not contracting, but they do discharge electrical impulses to the ventricles. However, no single impulse depolarizes the atria completely, so only an occasional impulse gets through the AV node. It is the most common sustained dysrhythmia in clinical practice. Atrial flutter (B) is a rapid atrial rhythm, but due to nodal delay, ventricular response rate is slower. Therefore, atrial flutter always occurs with some sort of AV block so that not all impulses are conducted. The resulting block is often variable (2:1, 3:1, 4:1). P waves have a characteristic sawtooth pattern. In normal sinus rhythm (C) and sinus tachycardia (D), the SA node is the pacemaker that causes the atria to depolarize regularly and, thus, the ventricles to depolarize regularly. Therefore, the ECGs for both of these rhythms have P waves and QRS complexes that occur regularly. The difference between these two rhythms is with the rate. The rate of sinus rhythm is 60-99. The rate of sinus tachycardia is >100. One Step Further Question: What is the INR range in a patient taking warfarin for atrial fibrillation? Reveal Answer: The INR range is 2-3. Rapid Review Rapid Review Atrial Fibrillation Alcohol Irregularly irregular No P waves Narrow QRS unless conduction block or accessory pathway Unstable: cardioversion Stable: Rate control with CCBs, ßBs <48 hours duration: cardiovert to sinus rhthym >48 hours duration: anticoagulate, echo to r/o thrombus, then cardioversion

An elderly patient is admitted to the hospital for multiple episodes of syncope. Telemetry reveals periods of sinus bradycardia in the 30s followed by a tachydysrhythmia with heart rates that fluctuate in the 100 to 160 range. During several of the bradycardia episodes the patient becomes hypotensive and symptomatic with shortness of breath, lightheadedness, and dizziness. Which of the following is the most likely underlying tachydysrhythmia associated with this patient's most likely diagnosis? A Atrial fibrillation B Atrial flutter C Multifocal atrial tachycardia D Sinus tachycardia

Correct Answer ( A ) Explanation: Atrial fibrillation is the most common tachydysrhythmia associated with sick sinus syndrome. Sick sinus syndrome is defined by electrocardiographic criteria because clinical signs and symptoms, if present, are highly variable. The electrocardiographic characteristics of the sick sinus syndrome include frequent periods of inappropriate, and often severe bradycardia, sinus pauses, arrest, and sinoatrial exit block, often without appropriate atrial and junctional escape rhythms. The failure of escape pacemakers may lead to symptoms including syncope. Alternating bradycardia and an atrial tachydysrhythmia is found in over 50 % of cases. Atrial fibrillation is most common, but atrial flutter and paroxysmal supraventricular tachycardias may also occur. The most common cause of sick sinus syndrome is the replacement of sinus node tissue by fibrous tissue. Symptoms of lightheadedness, presyncope, or syncope are often the reason that the patient seeks medical attention. Other manifestations include increasing dyspnea on exertion, worsening angina, and, in patients with alternating bradycardia and tachycardia, palpitations and other symptoms associated with a rapid heart rate. Atrial flutter (B), sinus tachycardia (D) and paroxysmal supraventricular tachycardia are also associated with sick sinus syndrome; however they are not as common as atrial fibrillation. Atrial flutter is a common dysrhythmia that is associated with similar conditions as atrial fibrillation and are sometimes co-existent. Sinus tachycardia is also associated with this sick sinus syndrome. However, the most common causes of sinus tachycardia are the normal response to exercise and conditions such as fright, anger, or stress. Multifocal atrial tachycardia (C) is not generally associated with sick sinus syndrome. It is defined as a heart rate over 100/min involving at least three distinct P-wave morphologies and is most commonly seen in those with significant lung disease.

A 22-year-old man presents to the Emergency Department after a syncopal episode. His ECG is shown above. He is currently asymptomatic with normal vital signs. Which of the following is the most appropriate next step in management? A Admit to a telemetry unit for cardiology consult and AICD placement B Discharge with cardiology follow up as an outpatient C Order emergent echocardiogram D Start amiodarone for treatment of presumed ventricular tachycardia

Correct Answer ( A ) Explanation: Brugada syndrome was initially described in 1992 and is an inherited disorder that is the result of a mutation in the cardiac sodium channel gene. Patients are predisposed to malignant dysrhythmias and sudden death. The mean age of sudden death is 41 years and males are significantly more likely than females to be affected. Most patients are asymptomatic and found only by incidental ECG. The type 1 ECG pattern characteristic of Brugada syndrome is coved ST segment elevation of greater than 2 mm in V1-V3. It can be transient and may be augmented by fever or pharmacologic stimulation (particularly sodium channel blockers). The type 1 pattern is considered diagnostic when combined with clinical criteria such as syncope, family history of early sudden death, characteristic ECG changes in family members, or history of ventricular fibrillation or ventricular tachycardia. The type 2 ECG pattern of saddleback ST segment elevation in V1-V3 is suggestive of Brugada syndrome but not diagnostic. Management involves admission to a telemetry unit for cardiology consultation. Definitive treatment is implantable cardioverter defibrillator placement. Patients with characteristic ECG changes and clinical criteria such as syncope are at a high risk for malignant dysrhythmias and sudden death and should not be discharged (B). There is no structural defect associated with Brugada syndrome so an echocardiogram (C) is not indicated in the the workup. Medications such as amiodarone (C) do not play a role in the management of Brugada syndrome. The only proven therapy to prevent sudden death in these patients is AICD placement.

An elderly man with congestive heart failure presents to the ED with a complaint of "not feeling normal." He is on several different medications. His ECG is shown above. Which of the following medications is the most likely cause of this ECG abnormality? Digoxin Furosemide Metoprolol Warfarin

Correct Answer ( A ) Explanation: Digoxin is a cardiac glycoside derived from the foxglove plant. It is used to increase the force of myocardial contraction in systolic heart failure and to decrease AV nodal conduction in atrial fibrillation. It works by inactivating the Na+ K+ ATPase pump on the cardiac cell membrane which leads to increased intracellular calcium and extracellular potassium. In individuals taking digoxin, there is a characteristic ECG pattern commonly referred to as the "digoxin effect." The presence of the "digoxin effect" on the ECG is not a marker of toxicity, rather it indicates that the patient is taking digoxin. The "digoxin effect" is characterized by (1) downsloping ST depression with a characteristic "slurred" appearance; (2) flattened, inverted, or biphasic T waves; (3) shortened QT interval. The overall morphology is commonly compared to the shape of a mustache. Furosemide (B) is a diuretic that is not directly related to changes in ECG morphology. However, in extreme cases it can lead to electrolyte abnormalities that are represented by ECG changes. Metoprolol (C) and other beta-blockers may cause a first-degree heart block (PR interval > 200 msec). Warfarin (D) does not affect the cardiac electrical pathway to produce ECG changes. One Step Further Question: What is digoxin used for? Answer: Its positive-inotropic and antidysrhythmic effects, at therapeutic doses, are used in treating atrial fibrillation and congestive heart failure. Rapid Review Digoxin Effect Not a marker of toxicity--only indicates patient is taking digoxin Downsloping ST depression with a characteristic slurred appearance ("Salvador Dali mustache") Flattened, inverted, or biphasic t waves Shortened QT interval

A 16-year-old man presents to the clinic for a sports physical. He never experiences any chest pain or shortness of breath and exercises frequently. An ECG is performed and is shown above. What is the most likely diagnosis? A First-degree atrioventricular block B Second-degree atrioventricular block (Mobitz Type II) C Sinus arrhythmia D Sinus bradycardia

Correct Answer ( A ) Explanation: First degree atrioventricular (AV) block is characterized by a lengthening of the PR interval > 200 msecs. There are regular conductions of the SA impulse to the ventricles represented by a QRS complex following each P wave. A normal PR interval is between 120 and 200 msec in length. This represents ventricular filling. First-degree AV block is the most common conduction disturbance and is typically seen in young or athletic individuals. It can also be seen in elderly individuals without heart disease or those with a previous history of myocardial infarction. It usually represents an incidental finding on an ECG and by itself does not cause any symptoms. There is no treatment necessary. Second degree AV block, or Mobitz Type II (B) also has a constant PR interval similar to first degree AV block; however, the PR interval can either be normal or prolonged. An infranodal block causes a widened QRS complex and an intermittently non-conducted P wave. Patients with Mobitz Type II AV block usually require a pacemaker as the block can progress to complete AV dissociation. Sinus arrhythmia (C) is characterized by the heart rate slightly increasing with inspiration and decreasing with expiration. It is frequently seen in young people with increased vagal tone and is typically a benign finding. Sinus bradycardia (D) has regular PR intervals and conduction of every atrial impulse similar to first degree AV block but is characterized by a bradycardic rate (heart rate < 60).

A 39-year-old woman presents with palpitations and lightheadedness. Her rhythm strip is shown above. Which of the following treatments is indicated? A Adenosine B Defibrillation C Procainamide D Sedation with etomidate following by electrical cardioversion

Correct Answer ( A ) Explanation: The electrocardiogram reveals a supraventricular tachycardia (SVT). Most cases of SVT result from sustained reentry occurring within the atrioventricular (AV) node, with a minority of cases of SVT resulting from a reentry loop from an ectopic atrial focus. The electrocardiographic hallmarks of SVT are a fast, regular rhythm with a narrow QRS complex. Since the depolarization does not come from the sinoatrial node, P waves do not precede each QRS complex, though P waves may be buried within or seen immediately before or after each QRS complex, known as "retrograde" P waves. Paroxysmal SVT is more common in females than males, with a peak incidence in the late teenage and young adult years. Most patients with SVT do not have underlying heart disease. Common symptoms include palpitations, lightheadedness, and dyspnea. Vagal maneuvers, such as carotid sinus massage and valsalva, are often successful in terminating SVT, especially early in the dysrhythmia course. If vagal maneuvers are ineffective, adenosine is the treatment of choice. The initial dose of adenosine is 6 mg rapid intravenous push, followed by a dose of 12 mg if the first dose is ineffective. Beta-blockers and calcium channel-blockers are alternative agents. Electrical cardioversion is used for refractory SVT or patients who are clinically unstable. Defibrillation (B) is the treatment for pulseless ventricular tachycardia and ventricular fibrillation. Sedation with etomidate following by cardioversion (C) is the treatment for refractory SVT or unstable patients. Procainamide (D) is not a first-line treatment for SVT.

A 59-year-old man presents complaining of a severe headache for three days that came on gradually, and is diffuse in nature. His blood pressure is 205/105 mm Hg. All other vital signs are normal. His physical exam and basic laboratory workup are negative. His electrocardiogram is shown above. Which of the following antihypertensives is contraindicated in this patient? Atenolol Enalapril Hydralazine Hydrochlorothiazide

Correct Answer ( A ) Explanation: Wolff-Parkinson-White syndrome (WPW) is characterized by an accessory conduction pathway that bypasses the AV node, resulting in a shortened PR interval and upsloping QRS wave, known as a delta wave. If conduction through the AV node is slowed or disrupted, patients are at risk for an accelerated tachyarrhythmia and sudden cardiac death. There are two types of WPW, each requiring different treatments. Orthodromic WPW is clinically indistinguishable from AVNRT and can be treated in a similar fashion. *Antidromic WPW is more dangerous as it can lead to tachyarrhythmias and death as above. Therefore, any medication that slows AV nodal conduction is contraindicated in antidromic WPW. These can be remembered by the mnemonic "ABCD"-- adenosine, beta-blockers, calcium channel blockers, and digoxin. Atenolol is a beta blocker and is therefore contraindicated.* Enalapril (B), an angiotensin-converting enzyme inhibitor, hydralazine (C), an arteriolar smooth muscle relaxant, and hydrochlorothiazide (D), a thiazide diuretic, do not interfere with AV nodal conduction and are therefore safe for use in patients with Wolff-Parkinson-White syndrome. One Step Further Question: Which tachydysrhythmia is most commonly seen in Wolff-Parkinson-White syndrome? Answer: Atrioventricular reentrant tachycardia (AVRT)

A 32-year-old previously healthy man presents to the ED with a 4-hour history of palpitations. He denies chest pain, shortness of breath, or history of similar palpitations. He does admit to heavy alcohol use in the past week, drinking 1 pint of vodka and a 24-pack of beer each day. In the ED, his vital signs are BP 135/75, HR 115, RR 14, and oxygen saturation 98% on room air. An irregularly irregular rhythm is heard on auscultation and an ECG shows atrial fibrillation. What is the next step in management? Chemical cardioversion Observation Rate control Synchronized cardioversion

Correct Answer ( B ) Explanation: This patient has holiday heart syndrome, which can produce atrial fibrillation, atrial flutter, or atrial tachycardia after excessive alcohol use. Patients generally present with palpitations. The rhythm tends to spontaneously convert back to a sinus rhythm within 24-48 hours; thus, the best step in management at this time is to observe the patient with cardiac monitoring. If the patient stays tachycardic beyond 24-48 hours, rate control (C) can be employed, using medications such as calcium channel blockers or beta-blockers. Chemical cardioversion (A) and synchronized cardioversion (D) are both unnecessary now, as the patient is likely to revert to a sinus rhythm on his own. However, if symptoms persist, cardioversion may be used to reduce the risk of thrombus formation in addition to the need for either anticoagulation or a transesophageal echocardiogram. In addition, if the patient becomes unstable at any time, synchronized cardioversion would be used.

Which of the following is an indication for permanent pacemaker placement? A Asymptomatic Mobitz type I second-degree AV block B Asymptomatic Mobitz type II second degree heart block C Asymptomatic sinus bradycardia with heart rate of 40/min D Asymptomatic three second sinus pauses

Correct Answer ( B ) Explanation: Asymptomatic Mobitz type II second-degree AV block is an indication for pacemaker placement. This block has a high risk of progressing to complete heart block and should be treated with pacemaker placement, regardless of symptoms. In general, the long term treatment for symptomatic sinus bradycardia or heart block without reversible cause is a permanent pacemaker. These devices are usually placed in the left pectoral area with leads inserted through a vein into the heart. Two general factors guide the decision to place a permanent pacemaker: the association of symptoms with a brady-dysrhythmia and the potential for progression of the rhythm disturbance. Progression is largely dependent on the anatomical location of the conduction abnormality. The location of an AV conduction abnormality, within the AV node or below the AV node in the His-Purkinje system is an important determinant of both the probability and progression rate of conduction system disease. Disease below the AV node, in the His-Purkinje system, is generally considered to be less stable. The most common indications for pacemaker implantation are sinus node dysfunction followed by AV block. Sinus bradycardia in which symptoms such as dizziness, lightheadedness, syncope, fatigue, or poor exercise tolerance are present should be treated. Acquired AV block is the second most common indication for permanent pacemaker placement. Complete, or third-degree AV block, advanced second-degree AV block, symptomatic Mobitz I or Mobitz II second-degree AV block are all indications for pacemaker placement. Asymptomatic sinus bradycardia with heart rate of 40/min (C), asymptomatic Mobitz type I second-degree AV block (A) and asymptomatic three second sinus pauses (D) are not indications for a pacemaker in the absence of symptoms.

Which of the following patients with atrial fibrillation has the greatest risk for complications from anticoagulation therapy? A A 64-year-old woman with hypertension and history of colon cancer B A 66-year-old man with a history of hypertension, diabetes, and ethanol abuse C A 79-year-old man with a history of congestive heart failure D A 90-year-old otherwise healthy woman

Correct Answer ( B ) Explanation: Atrial fibrillation is associated with arterial thromboembolism and ischemic stroke; therefore, anticoagulation is recommended in most cases. The HAS-BLED risk score is based upon seven risk factors for bleeding, including age > 65 years, and has been recommended within the European Society of Cardiology and Canadian guidelines for assessing the risk of bleeding in atrial fibrillation management. The 66-year-old man with a history of hypertension, diabetes, and ethanol abuse has a HAS-BLED score of 3 which corresponds to a high risk for bleeding. The 64-year-old woman with hypertension and history of colon cancer (A) has a score of 1. The 79-year-old man with a history of congestive heart failure (C) has a score of 1. The 90-year-old otherwise healthy woman (D) has a score of 1.

A patient presents with palpitations and dyspnea. She is placed on a cardiac monitor as seen (see image). Which of the following is the most likely diagnosis? A Atrial fibrillation B Atrial flutter C Ventricular fibrillation D Ventricular tachycardia

Correct Answer ( B ) Explanation: Atrial flutter is a dysrhythmia characterized by an atrial rate of 250-300 bpm. The most common site of pathology is a reentrant circuit in the right atrium about the tricuspid valve annulus. Etiologies include hypertensive heart disease, ischemic heart disease, rheumatic heart disease and cardiomyopathy. The resulting tachycardia typically causes decreased cardiac output with symptoms of palpitations, dyspnea, presyncope and fatigue. It is diagnosed electrocardiographically by regularly shaped and spaced P waves which have the appearance of a sawtooth pattern. These P waves will intermittently be followed by narrow QRS complexes. Management includes electrical or pharmaceutical cardioversion and ventricular rate control (calcium channel blockers and beta-blockers). Atrial fibrillation (A) is disorganized electrical activity and defined by an irregular rhythm with the absence of P waves. Ventricular fibrillation (C) produces a 'wavy line' ECG, of which no discernable rate, P wave or QRS complex is seen. This dyshythmia is associated with pulselessness. Ventricular tachycardia (D) is a wide complex tachycardia.

Which of the following best describes the finding seen in the ECG see image? A Atrioventricular block B Left bundle branch block C Right bundle branch block D Sinoatrial block

Correct Answer ( B ) Explanation: Bundle branch blocks are abnormal conduction abnormalities (not rhythm disturbances) in which the ventricles depolarize in sequence, rather than simultaneously, thus producing a wide QRS complex (> 120 msec) and a ST segment with a slope opposite that of the terminal half of the QRS complex. A left bundle branch block is a bifascicular block in which ventricular activation is by way of the right bundle branch. The impulse travels down the right bundle, activating the septum and the free wall of the right ventricle, and then continues on in the same direction to activate the free wall of the left ventricle. Because the dominant forces are traveling in the same direction, there is a tendency toward monophasic QRS complexes. The ECG in a LBBB will show a large wide R wave in lead I and a negative wave (QS or rS) in lead V1.

An 83-year-old is being evaluated in the emergency department after an episode of syncope. The woman was preparing dinner when she felt her heart start to race. The next thing she remembers is waking up on the floor. She experienced a similar episode about three weeks ago. She has never had anything like this before. Her past medical history is remarkable for hypertension, hyperlipidemia and hypothyroidism. Her medications include lisinopril, atorvastatin and levothyroxine. On physical exam her blood pressure is 142/83, heart rate 76/min, and respiration rate 13/min. Cardiac auscultation reveals no murmur. The remainder of her physical exam is normal. Electrocardiogram reveals normal sinus rhythm with left axis deviation. No cardiac rhythm abnormalities are detected. What is the most likely etiology of this patient's syncope? A Aortic stenosis B Cardiac dysrhythmia C Orthostatic hypotension D Vasovagal

Correct Answer ( B ) Explanation: Cardiac dysrhythmia is the most likely cause of this woman's syncope. Cardiac dysrhythmias are a common cause of syncope in the elderly population. It is characterized by a brief or absent prodrome and palpitations immediately preceding the event. Several episodes over a short period of time in someone with no history of syncope suggest a dysrhythmia. Given this patient's short prodrome, palpitations and history of a previous similar event makes a cardiac dysrhythmia the most likely etiology. Aortic stenosis (A) is unlikely the cause of her syncope. Aortic stenosis is associated with a crescendo-decrescendo systolic ejection murmur. Syncope related to aortic stenosis typically occurs during exertion and is associated with very severe disease. This patient's syncopal episode occurred while stationary. Additionally, she has no systemic symptoms of aortic stenosis. Vasovagal (D) is the most common cause of syncope in the general population. It is usually triggered by provoking factors such a blood draw or an intense emotion. Prodromal symptoms include feeling warm, sweating, nausea, and pallor. This woman does not report any of these symptoms. Orthostatic hypotension (C) causes syncope upon assuming an upright position from supine or sitting. It is often caused by hypovolemia, medications or autonomic nervous system disorders. This woman was standing while preparing dinner making orthostatic hypotension unlik

Which of the following is a class IC anti-arrhythmic? A Amiodarone B Flecainide C Lidocaine D Procainamide

Correct Answer ( B ) Explanation: Flecainide is a class IC anti-arrhythmic drug. All class I anti-arrhythmics exert their action at the fast sodium channels. The subsets of class I relate to how the drugs affect depolarization, repolarization and conduction. The class IC drugs slow depolarization and conduction. Flecainide is most commonly used in the treatment of supraventricular tachycardias as well as ventricular tachycardia not related to acute ischemia.

A 60-year-old woman with a history of hypertension, dyslipidemia and coronary artery disease was sent to the emergency department from her primary care physician's office for a heart rate of 40/min. She has no complaints except for mild fatigue. Her medications include metoprolol, atorvastatin, lisinopril and baby aspirin. Her ECG reveals sinus bradycardia and her physical exam is normal. Which of the following is the most appropriate next step in management? A Administer atropine B Make a medication adjustment C Schedule her for a temporary pacemaker D Watchful waiting

Correct Answer ( B ) Explanation: Medication adjustment is the most appropriate in this clinical situation. This patient has sinus bradycardia associated with mild fatigue and is hemodynamically stable. She is currently taking a beta blocker, metoprolol, which can cause or exacerbate sinus bradycardia. Sinus bradycardia is usually asymptomatic. However, symptoms related to the slow heart rate can occur, including lightheadedness, pre-syncope or syncope, as well as worsening of angina pectoris or heart failure. Symptoms may be subtle, with many patients noting only fatigue. This is frequently overlooked and ascribed to aging rather than bradycardia. The underlying cause of pathologic sinus bradycardia in most patients is fibrotic replacement of the sinus node associated with aging. Other causes include node damage secondary to infarction or cardiac surgery, infiltrative causes such as amyloid or sarcoidosis, increased vagal tone secondary to a Valsalva maneuver or vomiting, medications such as beta blockers or calcium channel blockers, and rarely genetic diseases. Sinus bradycardia is not necessarily pathologic; athletes and other highly conditioned persons may have heart rates of 40/min and sleeping heart rates of 30/min. Treatment is not indicated in asymptomatic patients with sinus bradycardia. When symptoms occur it is appropriate to search for a medication that may be depressing sinus node function. This patient is taking a beta blocker. This should be either be dose adjusted or discontinued before any other medications or more invasive approaches are initiated. Atropine (A) and temporary pacemakers (C) are used in ACLS protocol for bradycardia in patients who are hemodynamically unstable. This patient is hemodynamically stable with no lightheadedness, pre-syncope or syncope. Temporary or permanent electrical pacing may be required in acute or chronic symptomatic sinus bradycardia. Watchful waiting (D) is inappropriate as this patient has symptoms of fatigue.

A previously healthy 16-year old boy presents to your office after having a syncopal episode at the start of track practice. An ECG reveals a QTc of 520 ms. This is confirmed on a subsequent ECG. This finding is associated with which one of the following rhythm abnormalities? A Paroxysmal supraventricular tachycardia B Polymorphic ventricular tachycardia C Sinus arrest D Third degree atrioventricular block

Correct Answer ( B ) Explanation: Patients with repeated ECGs showing a QTc interval > 480 ms with a syncopal episode, or > 500 ms in the absence of symptoms, are diagnosed with long QT syndrome if no secondary cause such as medication use is present. Prolong QT interval is associated with polymorphic ventricular tachycardia, including torsades de pointes, and sudden cardiac death. Most cardiac events are precipitated by vigorous exercise or emotional stress, but they also can occur during sleep. Long QT syndrome is usually diagnosed after a person has a cardiac event such as syncope or cardiac arrest. In some situations, this condition is diagnosed after a family member suddenly dies. It may be treated with beta-blockers and implanted cardioverter defibrillators. Patients with long QT syndrome should avoid participation in competitive sports, strenuous exercise, and stress-related emotions. Epinephrine adrenaline for local anesthesia and asthma medication should be avoided in patients with long QT syndrome. Other medications that should be avoided include certain antibiotics, antifungals, antihistamines, antiarrythmics and psychotropic medications which prolong the QT interval. Third-degree atrioventricular block (D) results from various pathologic states causing infiltration, fibrosis, or loss of connection in portions of the healthy conduction system. Third-degree atrioventricular block can be either congenital or acquired. Paroxysmal supraventricular tachycardia (A) is a narrow-complex tachycardia that has a regular, rapid rhythm and is triggered by a reentry mechanism. This may be induced by premature atrial or ventricular ectopic beats. Other triggers include hyperthyroidism and stimulants, including caffeine, drugs, and alcohol. Paroxysmal supraventricular tachycardia is observed not only in healthy individuals; it is also common in patients with previous myocardial infarction, mitral valve prolapse, rheumatic heart disease, pericarditis, pneumonia, chronic lung disease, and current alcohol intoxication. Sinoatrial arrest (C) is when the sinoatrial node of the heart transiently ceases to generate the electrical impulses that normally stimulate the myocardial tissues to contract. It is defined as lasting from 2.0 seconds to several minutes. None of the above are associated with long QT syndrome. One Step Further Question: Which electrolyte abnormalities can cause a long QT syndrome? Answer: Hypokalemia and hypomagnesemia

A 55-year-old man presents after a syncopal event. He states he just started a new blood pressure medication. His heart rate is 41 beats/minute and his blood pressure is 95/60 mm Hg. Electrocardiogram shows sinus bradycardia. Which of the following medications should be administered? Adenosine Atropine Diltiazem Procainamide

Correct Answer ( B ) Explanation: Sinus bradycardia refers to a discharge rate from the sinoatrial node of < 60 beats/minute. Sinus bradycardia can be a result of pathologic factors like hypoxia, hypothermia, cardiac ischemia or infarction, hypothyroidism, or increased intracranial pressure. Many medications also cause sinus bradycardia, including beta-blockers, calcium-channel blockers, digoxin, and opioids. Sinus bradycardia may also be a normal finding in well-conditioned young people, athletes, during sleep, or as a result of vagal stimulation. On electrocardiogram, sinus bradycardia is indistinguishable from sinus rhythm other than having a slower rate. Patients with sinus bradycardia may be asymptomatic or may complain of dizziness or lightheadedness. An especially slow rate may result in signs of hypoperfusion (e.g. hypotension, altered mental status, or ischemic chest pain). The treatment of sinus bradycardia depends on the underlying cause and the clinical effects. Underlying causes should be corrected. Unstable patients should be treated with atropine while transcutaneous pacing is initiated and arrangements for transvenous pacing are made. Infusions of dopamine or epinephrine are also indicated to increase the heart rate if atropine is ineffective. Glucagon is used to treat cardiotoxicity from beta-blocker or calcium channel blocker overdose. Adenosine (A) is an atrioventricular (AV) nodal blocker used in the treatment of supraventricular tachycardias. Diltiazem (C) is a calcium channel blocker used in the treatment of tachydysrhythmias. Procainamide (D) is an antiarrhythmic used in the treatment of tachydysrhythmias.

A 35-year-old man presents to the emergency department after a motor vehicle collision with prolonged extrication. He complains of severe pain in his leg, which was trapped under the car for 3 hours. A 12-lead ECG obtained in triage is shown above. Which of the following interventions will decrease total body potassium stores? A Calcium gluconate B Cation-exchange resins C Insulin D Magnesium sulfate

Correct Answer ( B ) Explanation: Symptomatic hyperkalemia is a life-threatening electrolyte abnormality typically seen in patients with underlying acute or chronic kidney disease. It can also be seen in conditions that cause increased tissue breakdown such as tumor lysis syndrome, rhabdomyolysis, and crush injuries. Muscle weakness and paralysis, cardiac conduction abnormalities and cardiac dysrhythmias are the most serious manifestations of hyperkalemia.Treatment of hyperkalemia includes antagonizing the membrane effects of potassium, driving extracellular potassium into cells, and removing potassium from the body. Calcium directly antagonizes the effect of potassium on cell membrane excitability, decreasing the cardiotoxic effects of hyperkalemia. While immediate administration of calcium gluconate or calcium chloride does not alter potassium levels, calcium helps prevent potentially fatal cardiac conduction abnormalities or dysrhythmias until the excess extracellular potassium can be driven into cells or out of the body. Treatment can then be initiated with agents to drive excess potassium intracellularly, including insulin (with glucose), beta-2 agonists such as albuterol and sodium bicarbonate. These treatments act to quickly decrease the level of extracellular potassium until treatment such as dialysis, loop or thiazide diuretics, or cation-exchange resins can be initiated to remove excess potassium from the body. Calcium gluconate (A) stabilizes the cardiac membrane, antagonizing the effects of potassium on the heart. While it does not affect the amount of extracellular or total body potassium, it prevents possibly fatal cardiac dysrhythmias until definitive treatment for hyperkalemia can be initiated. Insulin (C) causes an intracellular shift of potassium, lowering the extracellular potassium concentration. It does not, however, alter total body potassium levels. Magnesium sulfate (D) is not used in the treatment of hyperkalemia.

Which of the following agents is first line for rate control in atrial fibrillation with rapid ventricular response in the setting of compensated systolic heart failure? A Amiodarone B Carvedilol C Digoxin D Diltiazem

Correct Answer ( B ) Explanation: The beta blocker, carvedilol, is a first line agent in rate control of atrial fibrillation with rapid ventricular response in the setting of compensated systolic heart failure. Other first line beta blockers include extended release metoprolol succinate and bisoprolol. Although beta blockers do not appear to improve mortality in the setting of heart failure and atrial fibrillation, the rationale behind using them is that there is no evidence of harm with their use. Additionally, the alternatives to beta blockers have significant limitations. Alternatives include calcium channel blockers which have been associated with a greater mortality, digoxin which has a lesser efficacy and amiodarone which has more side effects. Use of beta blockers in patients with heart failure due to systolic dysfunction in the absence of atrial fibrillation, have been shown to reduce hospitalizations and improve survival. So when choosing a rate control agent it is advised to use one with a mortality benefit for concomitant systolic heart failure. Non-dihydropyridine calcium channel blockers such as verapamil and diltiazem (D) are inappropriate as these agents should be avoided in those with a reduced left ventricular systolic function. Digoxin (C) may be considered as second line in patients who cannot receive a beta blocker for rate control. In the event that adequate rate control cannot be achieved with either a beta blocker or a combination of a beta blocker and digoxin, then amiodarone (A) may be considered. Amiodarone can be used alone or in combination with other rate-slowing agents. Amiodarone is not recommended as a chronic rate-control medication, but in the acute setting can assist with rate control in a patient who cannot tolerate other therapies.

A 61-year-old woman presents to the Emergency Department after several episodes of syncope. Her rhythm strip is shown above. Which of the following atrioventricular blocks is present? A First-degree AV block B Second-degree Mobitz type I AV block C Second-degree Mobitz type II AV block D Third-degree AV block

Correct Answer ( B ) Explanation: The patient has a second-degree Mobitz type I AV block. A first-degree AV block refers to slowed conduction through the AV node, resulting in a prolonged PR interval, but preserved conduction of each sinus-generated impulse. In second-degree AV block, there is intermittent conduction through the AV node—some atrial impulses are conducted through to the ventricles while others are blocked. There are two types of second-degree block, Mobitz type I and type II. In second-degree Mobitz type I block, also referred to as Wenckebach, there is progressive delay in each AV conduction until AV conduction is finally blocked. On electrocardiogram, this appears as progressive lengthening of each PR interval until a QRS complex is eventually dropped and the cycle repeats. Grouped beating, or clustering of QRS complexes separated by a pause from the dropped beat, is characteristic of second degree type I AV block. Mobitz type I block is often transient, seen in association with inferior wall myocardial ischemia, medication toxicity, or cardiac surgery. Treatment is usually not necessary unless it is associated with a very slow rate with signs of hypoperfusion. In which case atropine, which increases the rate of sinus node firing and conduction through the AV node, is usually effective. The other type of second-degree AV block, Mobitz type II, is more ominous. Type II block is characterized by a constant PR interval with periodic non-conducted impulses, seen as absence of a QRS complex. Because Mobitz type II blocks are associated with progression to third-degree, or complete heart block, patients should have transcutaneous pacer pads placed in anticipation of possible need. In third-degree AV block, there is no conduction through the atrioventricular node, and an escape pacemaker is responsible for the ventricular rate. On electrocardiogram, there is no association of P waves with QRS complexes, and the ventricular rate is 40-60 beats/minute, depending on whether a junctional or ventricular escape pacemaker takes over. Pacing is usually needed in complete heart block. First-degree AV block (A) shows prolonged PR interval, but no dropped beats. Second-degree Mobitz type II AV block (C) is characterized by a constant PR interval with sporadic dropped beats. Third-degree AV block (D) is characterized by no association between P waves and QRS complexes.

A 58-year old man is brought to the ED for chest pain that started 30 minutes prior to arrival while he was jogging in the park. Initially, the patient's cardiac monitor shows sinus tachycardia with a rate of 120 beats per minute. However, while you are interviewing the patient in the resuscitation bay, he suddenly becomes pale, *pulseless*, and the above rhythm is seen on the cardiac monitor. Which of the following is the definitive next step to manage this rhythm? A Chest compressions B Defibrillation C Epinephrine D Synchronized cardioversion

Correct Answer ( B ) Explanation: The treatment of ventricular fibrillation depends upon whether the onset was witnessed or unwitnessed. If the cardiac arrest is witnessed (as in this case) and of short duration with an initial rhythm of ventricular fibrillation or tachycardia, the patient should receive immediate defibrillation with 200 joules biphasic (or 360 joules monophasic). Newer biphasic defibrillators are preferred. They have a better first-shock success rate than the older monophasic models have while delivering less electrical current and causing less myocardial cell damage. However, if the downtime is unknown, two minutes of CPR (A) is recommended prior to defibrillation for a shockable rhythm to help "prime" the heart to receive the shock. The main benefit of epinephrine (C) during CPR is derived from its alpha-receptors activity, which produces a direct increase in peripheral vascular resistance. In turn, this leads to a subsequent rise in central aortic blood pressure and an improvement in coronary perfusion pressure during chest compressions. For patients with a shockable rhythm, defibrillation is priority, and while epinephrine may be helpful, it is likely not the definitive step in management.. After the initial dose, epinephrine can be readministered every three to five minutes. Synchronized cardioversion (D) is reserved for those patients with a palpable pulse. *This pt was pulseless* !!!

A 52-year-old man presents from his primary care physician's office for evaluation of an abnormal electrocardiogram. His ECG is seen above. He has no symptoms. What is the appropriate intervention? A Measurement of cardiac enzymes B No intervention C Telemetry observation D Transcutaneous pacer pad placement

Correct Answer ( B ) Explanation: This ECG demonstrates an example of Type 1 second-degree atrioventricular (AV) block. AV block results from impaired conduction through some portion of the electrical circuit impairing communication between the atria and ventricles. In first and second degree AV block there is a partial disruption of the electrical circuitry as opposed to third degree heart block when there is no electrical communication between the atria and ventricles (AV dissociation). In second-degree heart block, some sinus impulses do not reach the ventricles at all. In type 1 second-degree AV block, there is lengthening of the PR interval until ultimately a beat is dropped. This is also known as Wenckebach or Mobitz I. In many cases, type 1 second-degree heart block is a normal variant. Other times, the etiology is likely related to increased vagal tone and in most cases requires no treatment. It can also occur in a myocardial infarction and usually resolves after the infarct period. Type 2 second-degree heart block is characterized by a dropped beat without any prolongation of the PR interval. Type 2 is never considered a normal variant. In the absence of any cardiac symptoms, the measurement of cardiac enzymes (A) in type 1 second-degree heart block is not necessary. Telemetry observation (C) is not required in type 1 second-degree heart block because it does not typically progress into other forms of heart block. In patients with type 2 heart block, this interruption of the electrical pathway occurs below the level of the AV node and can progress. Transcutaneous pacer pads (D) are often placed on the chest of a patient with complete heart block (type 3) because of the high chance for impaired cardiac output. These patients will require pacemaker placement if the heart block is not due to a reversible cause (e.g. hyperkalemia).

What is the most likely underlying chronic medical problem in the patient with the following ECG? A Cardiomyopathy B COPD C Hyperthyroidism D Mitral stenosis

Correct Answer ( B ) Explanation: This electrocardiogram demonstrates multifocal atrial tachycardia, a form of atrial tachycardia diagnosed on the electrocardiogram by three distinct p-wave morphologies. In approximately 60% of cases, patients have underlying pulmonary disease, most commonly COPD. Patients may have a primary cardiac pathology although this is much less common. Cardiomyopathy (A) is not commonly associated with atrial dysrhythmias, but may be the causative agent of ventricular dysrhythmias, particularly ventricular tachycardia. Hyperthyroidism (C) is on the differential diagnosis of atrial fibrillation and should be excluded in cases of new onset atrial fibrillation. Valvular disease, particularly mitral stenosis (D) may also lead to atrial fibrillation as the atrium dilates over time as a result of the stenosis. One Step Further Question: What is the treatment for multifocal atrial tachycardia? Answer: Most commonly, treat the underlying cause which will often improve the dysrhythmia. Nodal agents are not as effective as in other dysrhythmias. Rapid Review Multifocal Atrial Tachycardia Irregularly irregular ≥ 3 different P waves Rate: 100-180 Nonconducted P waves are present Pulmonary disease/COPD Treat underlying condition Symptomatic: CCBs for rate control

A 72-year-old man presents for evaluation of palpitations. He has a regular, wide complex tachycardia at a rate of 140 bpm. Which of the following supports a diagnosis of ventricular tachycardia? A Discordance of the QRS axis in the precordial leads B Fusion beats C Leftward axis D ST elevation greater than 5 mm

Correct Answer ( B ) Explanation: Ventricular tachycardia (VT) originates from a location within the ventricle typically outside of the normal conduction system. Increased automaticity or a reentry circuit may cause it. The majority of patients with VT have underlying cardiac disease. The most common form of VT is monomorphic where the QRS complexes appear the same morphologically. Since the electrical impulse originates outside of the conduction system, the QRS complex is wide. It is sometimes challenging to differentiate between VT and other wide complex tachycardias (eg, supraventricular tachycardia with aberrancy). VT is typically regular although there may be a small amount of irregularity helping to distinguish between it and supraventricular tachycardia. Fusion beats occur when impulses from two different locations (one within the ventricle and one in a supraventricular location) activate the ventricle. The result is a QRS complex with morphology resembling a hybrid of a sinus beat and intraventricular beat. These are diagnostic of VT because they represent AV dissociation. Capture beats occur when a sinus beat is normally conducted and a single beat with the sinus QRS morphology occurs within a wide complex tachycardia. Discordance of the QRS axis in the precordial leads (A) is NOT suggestive of VT. Typically across the precordium in VT the QRS complex demonstrates concordance or its polarity. A leftward axis (C) is not suggestive of VT and a more characteristic axis deviation is right superior. ST elevation greater than 5 mm (D) is not part any diagnostic criteria for the identification of VT.

A 37-year-old woman with a history of Wolff-Parkinson-White presents to the emergency department with shortness of breath and lightheadedness. Her vital signs on arrival are T 36.9°C, HR 160, BP 80/50, RR 27. Her ECG reveals a narrow complex regular tachycardia. Which of the following is the most appropriate next step in the management of this patient? Adenosine Cardioversion Fluid bolus Procainamide

Correct Answer ( B ) Explanation: Wolff-Parkinson-White (WPW) syndrome is a ventricular preexcitation syndrome that involves an anatomic accessory pathway that bypasses the AV node and forms a direct electrical connection between the atria and ventricle, placing patients at high risk of supraventricular tachydysrhythmias. Tachydysrhythmias in WPW should be treated according to the patient's clinical stability, with hemodynamically unstable patients receiving immediate electrical cardioversion. The first intervention in orthodromic AVRT in a hemodynamically stable patient is a vagal maneuver. If this fails, intravenous adenosine can be used, followed by AV blocking agents such as beta-blockers or calcium channel blockers. Definitive treatment of WPW syndrome is ablation of the accessory pathway. Adenosine (A) is the first line pharmacologic intervention in hemodynamically stable orthodromic AVRT in whom vagal maneuvers have failed. It is not appropriate in a symptomatic, hemodynamically unstable patient. Administering a fluid bolus (C) is appropriate treatment of hypotension in a patient with hypovolemic or distributive shock but would not be appropriate for a patient that is hypotensive due to a dysrhythmia. Procainamide (D) is the treatment of choice in hemodynamically stable antidromic AVRT and is third line for hemodynamically stable orthodromic AVRT. It should not be used in hemodynamically unstable patients.

An 18-year-old man with a history of Wolff-Parkinson-White presents with a one hour history of palpitations. His triage ECG is shown above. Which of the following is the most likely diagnosis? A Antidromic atrioventricular reciprocating tachycardia B Atrial fibrillation C Orthodromic atrioventricular reciprocating tachycardia D Ventricular tachycardia

Correct Answer ( B ) Explanation: Wolff-Parkinson-White (WPW) syndrome is a ventricular preexcitation syndrome that involves an anatomic accessory pathway that bypasses the AV node and forms a direct electrical connection between the atria and ventricles. Patients with this accessory pathway are at high risk of supraventricular tachydysrhythmias, the most common of which is atrioventricular reentrant (or reciprocating) tachycardia (AVRT). Atrial fibrillation is also common and is seen in approximately 25% of symptomatic WPW tachydysrhythmias. Atrial fibrillation in WPW has a very rapid ventricular rate due to the ability of the accessory pathway to conduct the impulse faster than the AV node. It is usually a wide-complex tachycardia due to impulses conducted through both pathways fusing to form a depolarization. QRS morphology in atrial fibrillation with underlying WPW typically varies from beat to beat due to variable conduction through the accessory tract versus the AV node. Antidromic AVRT (B) occurs due to anterograde conduction of the atrial impulse through the accessory pathway and return of the impulse through the atrioventricular node. This results in a wide complex regular tachycardia that is often confused with a monomorphic ventricular tachycardia. Orthodromic AVRT (C) is the most common symptomatic tachydysrhythmia seen in WPW and results from anterograde conduction of the atrial impulse through the AV node with return of the impulse back through the accessory pathway. It is a narrow complex tachydysrhythmia and is difficult to distinguish from a typical supraventricular tachycardia. Ventricular tachycardia (D) can be distinguished from atrial fibrillation with underlying WPW due to atrial fibrillation being faster, irregular, and having a variable QRS morphology. One Step Further Question: What is the treatment of choice of unstable atrial fibrillation in WPW syndrome? Answer: Cardioversion. Rapid Review Wolff-Parkinson-White (WPW) Syndrome ECG will show short PR interval, delta wave, wide QRS Most commonly caused by an accessory pathway (bundle of Kent) connects atria to ventricles, bypassing AV node Definitive treatment is radiofrequency ablation

Which of the following describes ECG findings in Wolff-Parkinson-White Syndrome? The delta wave reflects rapid conduction through the AV node The PR interval is shortened in sinus rhythm The QRS complex is typically narrow in sinus The QT segment is prolonged in sinus rhythm

Correct Answer ( B ) Explanation: Wolff-Parkinson-White (WPW) syndrome is a ventricular preexcitation syndrome that involves an anatomic accessory pathway that bypasses the AV node and forms a direct electrical connection between the atria and ventricles. Patients with this accessory pathway are at high risk of supraventricular tachydysrhythmias. The classic triad of WPW syndrome can be seen when the patients are in sinus rhythm: (1) a shortened PR interval (2) a fused QRS complex and (3) a delta wave at the beginning of the QRS complex. These classic signs are typically not seen when the patient is in a tachydysrhythmia. The shortened PR interval is due to rapid conduction of the atrial impulse through the accessory tract, which is able to conduct the impulse faster than the physiologic pace of the AV node. The fused, widened QRS complex results from activation of the ventricles by both the physiologic AV node and the aberrant impulse from the accessory tract. The initial part of this widened complex, called the delta wave, represents ventricular activation by the impulse conducted through the accessory pathway while the remainder of the QRS complex represents normal ventricular depolarization from the impulse travelling through the AV node. The classic delta wave of WPW represents conduction through the accessory tract, not the AV node (A). The QRS complex (C) is typically fused or widened due to ventricular activation by both the accessory pathway and the AV node. The QT interval (D) is typically unchanged in WPW syndrome. One Step Further Question: What is the most common tachydysrhythmia seen in Wolff-Parkinson-White syndrome? Answer: Orthodromic atrioventricular reentrant tachycardia.

A man who presents with syncope is placed on the cardiac monitor. On the monitor you note a repeating trend of 6 P waves, 5 of which are followed by a narrow QRS complex and 1 of which is not followed by a QRS complex. The PR interval during this trend progressively increases. Which of the following is the most likely diagnosis? A First-degree AV block B Third-degree AV block C Type I second-degree AV block D Type II second-degree AV block

Correct Answer ( C ) Explanation: A key distinction between first-degree and second-degree heart block is that in first-degree block the P wave is always followed by a QRS complex. In other words, the ratio of P waves to QRS complexes is 1:1, or, the electrical signal from the atria always passes to the ventricles. In second-degree AV block, the electrical impulse sometimes gets to the ventricles. There are two main types of second-degree AV block. In Mobitz type I, or Wenckebach, second-degree block, there is a progressive beat-to-beat lengthening of the PR interval until a P wave does not conduct through the AV node. The absent conduction and resultant "missing" QRS complex is called a "dropped" QRS, which represents an absent beat of ventricular contraction. First-degree AV block (A) has a 1:1 ratio of P waves to QRS complexes. Mobitz type II second-degree heart block (D) is characterized by a nonconducted P wave which is not preceded by progressive PR interval prolongation. AV dissociation, or third-degree AV block (B), occurs when none of the P waves conduct through the AV node. This complete AV block occurs with separate atrial and ventricular rates. There is no discrete correlation or trend between P waves and QRS complexes.

Which of the following medications can be used in the treatment of hemodynamically stable atrial fibrillation with concomitant Wolff-Parkinson-White syndrome? A Adenosine B Diltiazem C Ibutilide D Metoprolol

Correct Answer ( C ) Explanation: Ibutilide can be used for rhythm control in hemodynamically stable atrial fibrillation with concomitant Wolff-Parkinson-White syndrome, which is also referred to as preexcited atrial fibrillation. Atrial fibrillation can occur in up to one third of patients with Wolff-Parkinson-White syndrome. Tachydysrhythmias can be facilitated by direct conduction from the atria to the ventricles via the accessory pathway, bypassing the AV node, as seen with atrial fibrillation or atrial flutter in conjunction with Wolff-Parkinson-White. Treatment with AV nodal blocking drugs such as adenosine, calcium-channel blockers, beta-blockers and digoxin may increase conduction via the accessory pathway with a resultant increase in ventricular rate and possible degeneration into ventricular tachycardia or ventricular fibrillation. The goals of acute drug therapy for preexcited atrial fibrillation are prompt control of the ventricular response and, ideally, termination of atrial fibrillation. If the patient is unstable because of a rapid ventricular response, electrical cardioversion should be performed. For more stable patients, trials of intravenous medications can be performed cautiously. Treatment of preexcited atrial fibrillation requires a parenteral drug that lengthens antegrade refractoriness and slows conduction in both the AV node and the His-Purkinje system as well as the accessory pathway. Ibutilide is a class III antiarrhythmic drug that prolongs the refractoriness of the AV node, His-Purkinje system, and accessory pathway, and is useful for termination of atrial fibrillation and atrial flutter. If ibutilide is not available, or if the concern for drug-induced QT prolongation and polymorphic ventricular tachycardia is high, procainamide in another option. *Adenosine (A), Diltiazem (B), and Metoprolol (D) are all AV nodal blocking agents and should be avoided in patients with preexcited atrial fibrillation. Blocking the AV node can promote and enhance conduction down the accessory pathway, which can deteriorate into an unstable rhythm such as ventricular tachycardia or ventricular fibrillation.*

A 44-year-old woman presents to the Emergency Department with palpitations following the death of her mother 2 days prior. Her ECG is consistent with normal sinus rhythm with a QTc interval of 520 msec. Her heart rate is 86 and blood pressure is 117/82. Her medications include lisinopril and metformin. She took lorazepam yesterday for anxiety related to the loss of her mother. What is the most appropriate therapy? A Administer adenosine B Discontinue lorazepam C Initiate metoprolol D Perform vasovagal maneuvers

Correct Answer ( C ) Explanation: Long QT syndrome is defined by a QT interval of more than 460 msec for women and more than 440 msec for men on ECG. It is more commonly caused by medications than by familial prolongation. Physical and emotional stress are common triggers. Manifestations include palpitations, syncope and cardiac arrest. Beta-blockade with medications such as metoprolol may be appropriate in patients who are hemodynamically stable. Cardiology consult or referral is recommended because implantation of a cardiac defibrillator can be beneficial for those at risk of sudden cardiac death. Adenosine (A) is recommended in patients with palpitations due to supraventricular tachycardia. Lorazepam (B), however, is not associated with QT prolongation. Vasovagal maneuvers (D) are recommended in patients with palpitations due to supraventricular tachycardia.

A 73-year-old man with a history of hypertension and COPD presents with the ECG seen above. Which of the following is the correct diagnosis? A Atrial fibrillation B Atrial flutter C Multifocal atrial tachycardia D Wandering pacemaker

Correct Answer ( C ) Explanation: Multifocal atrial tachycardia (MAT) is a subset of atrial tachycardia, with more than 2 foci of impulse formation. On the ECG, at least 3 distinctively different P waves with varying P'-R, R-R, and P'-P' intervals are seen. MAT is often associated with pulmonary disease (e.g., COPD) and hypoxemia, either directly from these conditions or as a result of beta-adrenergic agonist or chronic methylxanthine treatment. MAT often resolves when the underlying condition is treated. MAT is easily confused for atrial fibrillation because both rhythms may be irregular and tachycardic, but P waves exist in MAT, not atrial fibrillation. Atrial fibrillation (A) is characterized as an irregularly irregular rhythm where there are multiple atrial foci firing rapidly and continuously, creating a fibrillating baseline. The QRS complexes are occurring irregularly but are usually of normal duration and configuration. The ECG above has P waves, which are not present in atrial fibrillation. Atrial flutter (B) is a fast atrial tachycardia (with an atrial rate upwards of 250-300 b/min) that can be due to a re-entry problem and is characterized by a sawtooth pattern (created by the rapidly firing atrial foci); the QRS complexes are typically regular. An irregular rhythm can occur with variable blocks. Wandering pacemaker (D) is similar to MAT in that there are 3 different P wave morphologies However, the rate in wandering pacemaker is 60-100 beats per minute, slower than MAT.

A 35-year-old woman with alcohol abuse presents to clinic complaining of "skipped heart beats." Her examination is normal. An in-office ECG is also normal. Laboratory testing is nonspecific. Which of the following is the next most appropriate test? A Echocardiogram B Electromyography C Holter monitor D Sestamibi scan

Correct Answer ( C ) Explanation: Premature atrial contractions (PACs) are extra beats that originate outside the sinus node from ectopic atrial pacemakers. They appear interspersed throughout an underlying rhythm, usually sinus. Common symptoms include palpitations, described by many as a skipped beat or a missing beat. Others describe it as a stopping or flipping of the heartbeat. These symptoms mimic those of premature ventricular contractions. Causes include psychological stress, hypertension, valvulopathy, ischemic cardiac disease, stimulants (caffeine, tobacco, alcohol), digitalis toxicity, electrolyte abnormalities, and idiopathic. Most cases are asymptomatic and occur in otherwise healthy hearts. Many times one can only find them with continuous 24 hour monitoring via a Holter monitor. PACs are benign, however, a small amount may deteriorate into atrial flutter or fibrillation as well as supraventricular tachycardia. Although echocardiography (A) is usually part of the evaluation of PACs, one must first diagnose them by finding them on electrocardiography. Electromyography (B) is used in the evaluation of nerve and skeletal muscle disorders, not cardiac disorders. Sestamibi scanning (D) is a nuclear medicine test used in the evaluation of myocardial ischemia and infarction. It is not the next most appropriate test when the clinician does not have a working diagnosis of "skipped beats." One Step Further Question: What is the medical treatment of symptomatic premature atrial contractions? Reveal Answer:Calcium-channel blockers or beta-blockers. Rapid Review Premature Atrial Contractions Extra beats that originate outside sinus node Originate from ectopic atrial pacemaker Different morphology from sinus P waves Appears interspersed throughout an underlying rhythm

Which of the following is a medical treatment for congenital long QT syndrome? A Flecainide B Propafenone C Propranolol D Quinidine

Correct Answer ( C ) Explanation: Propranolol is the appropriate treatment for congenital long QT syndrome as beta blockers are the mainstay of treatment for this condition. Long QT syndrome is the most common channelopathy and is thought to affect 1 in 5000 persons. It is a disorder of myocardial repolarization characterized by a prolonged QT interval on ECG and an increased risk of sudden cardiac death: QTc is usually > 460 ms. This syndrome is associated with an increased risk of a characteristic life threatening polymorphic ventricular tachycardia known as torsades de pointes or "twisting of the points." The primary symptoms in patients with long QT syndrome include palpitations, syncope, seizures, and cardiac arrest, and patients usually have syncope related to polymorphic ventricular tachycardia. Factors conferring the highest risk for sudden cardiac death include a history of sudden cardiac arrest, recent syncope, and QTc interval greater than 500ms. Beta blockers, such as propranolol, have shown to reduce both syncope and sudden cardiac death. The goal of beta blocker therapy is to blunt the maximal heart rate achieved during exertion and are thought to interrupt the "trigger" for torsades de pointe and shorten the QT interval.

A 71-year-old woman presents with 2 days of dizziness and "almost passing out." Her ECG shows episodes of alternating bradycardia and tachycardia with narrow QRS complexes. Which of the following is the most likely diagnosis? Atrial flutter Digitalis toxicity Sick sinus syndrome Ventricular tachycardia

Correct Answer ( C ) Explanation: Sick sinus syndrome occurs as a result of disease of the sinoatrial (SA) node. It is associated with tachycardia-bradycardia syndrome in which the sinus rate varies from fast to slow and back again. ECG shows an irregular rhythm with pauses in sinus activity. Management depends on the presentation. For rhythms that are too slow, the patient may require a pacemaker. Whereas rhythms that are too fast may be treated with calcium channel blockers or beta-blockers. Atrial flutter (A) is associated with regular tachycardia and saw tooth appearance of P waves. Digitalis toxicity (B) is associated with a high degree AV block. Ventricular tachycardia (D) is not associated with a bradycardia. One Step Further Question: Which medications are associated with periodic sinus arrest? Answer: Digitalis, procainamide and quinidine.

A 23-year-old woman presents with acute onset dizziness and palpitations. Her ECG reveals evidence of AV nodal reentrant tachycardia with a rate of 170. Her blood pressure is 140/70 mm Hg. Which of the following is the most appropriate initial treatment? A Atropine B Cardioversion C Valsalva maneuver D Verapamil

Correct Answer ( C ) Explanation: Supraventricular tachycardias (SVT) include paroxysmal, reentry or preexcitation tachycardias. Reentry SVTs include AV nodal reentry (AVNRT), atrioventricular reentry, or atrial reentry. Reentry circuits require the presence of at least two different conduction pathways with differential refractory times. It is characterized by an abrupt onset and termination of tachycardia, that distinguishes it from sinus tachycardia, which has gradual changes in rate. It is precipitated by a premature atrial or ventricular contraction or hyperadrenergic state. The ECG shows a regular, fast rhythm with absent P waves and narrow QRS complex. Unstable patients require immediate synchronized cardioversion. Stable patients, such as the patient above, should first undergo vagal maneuvers. Some common vagal maneuvers include holding your breath and bearing down (Valsalva maneuver), coughing, gagging, and immersing your face in ice-cold water. If vagal maneuvers are unsuccessful, adenosine is used both diagnostically and therapeutically. Adenosine transiently blocks the AV-node and allows the circuit to "reset." Atropine (A) is used to treat bradydysrhythmias. It inhibits vagal activity to the heart which leads to an increase in heart rate. Cardioversion (B) is not indicated in hemodynamically stable patients with PSVT. However, if unstable, the patient should undergo cardioversion. If vagal maneuvers are unsuccessful, intravenous adenosine or calcium-channel blockers, such as verapamil (D), are first-line pharmacologic options, which block the AV-nod

A patient is being evaluated for palpitations in the emergency department. An ECG reveals a short PR interval, widened QRS and slurred upstroke of the QRS complex. Which of the following is the name of bypass pathway associated with this condition? A Bachmann's bundle B Bundle of His C Bundle of Kent D Left bundle branch

Correct Answer ( C ) Explanation: The bundle of Kent is the bypass pathway associated with Wolff-Parkinson-White syndrome, which is the most likely diagnosis in this patient. Wolff-Parkinson-White syndrome is a pattern of pre-excitation on ECG accompanied by a symptomatic tachycardia. It results from an accessory pathway that directly connects the atria and ventricles and bypasses the AV node. The bypass pathways are accessory connections called Kent bundles between the atrium and ventricle. If a bypass pathway conducts antegrade, a pre-excitation pattern may be seen on the ECG. This includes a short PR interval because the bypass pathway conducts rapidly and faster than the AV node; a delta wave, which is a slurred initial segment of the QRS complex resulting from slow conduction through ventricular tissue instead of the His-Purkinje system; and a widened QRS that consists of fusion between early ventricular activation from pre-excitation with the later ventricular activation resulting from transmission through the AV node.

An 85-year-old woman presents with generalized weakness for five days. Her vital signs are unremarkable except for a HR of 130 bpm. Her ECG is shown above. Which of the following represents the first-line treatment for this patient? Adenosine Amiodarone Diltiazem-CCB Synchronized cardioversion

Correct Answer ( C ) Explanation: The patient has atrial fibrillation with rapid ventricular response. Treatment of atrial fibrillation starts by distinguishing it from other chaotic rhythms (e.g., multifocal atrial tachycardia) and recognition of any underlying causes. Asymptomatic atrial fibrillation at a rate of <100 bpm requires no specific emergent therapy. Stable patients who have a rapid ventricular response (>100 bpm) should receive rate or rhythm control. Because the onset of atrial fibrillation in this patient is unknown, rhythm control in the ED is contraindicated. Intravenous calcium channel blockers (diltiazem, verapamil) or beta-adrenergic blockers (metoprolol) are first-line rate-controlling agents for stable atrial fibrillation. Adenosine (A) is a short-acting AV-nodal blocking agent used in the treatment of AV-nodal re-entry tachycardia (AVNRT), not atrial fibrillation. Amiodarone (B) is a class III antidysrhythmic that is used for the treatment of both ventricular and supraventricular dysrhythmias. It has the ability to cardiovert an irregular rhythm (atrial fibrillation) and should be avoided in this patient. In addition, amiodarone has an unfavorable side effect profile compared to calcium channel blockers. Synchronized cardioversion (D) should not be performed for atrial fibrillation when the exact onset is unknown. Patients should undergo an echocardiogram to evaluate for an atrial thrombus prior to cardioversion. However, in the unstable patient, the benefit of cardioversion outweighs the risk of thromboembolism, and cardioversion is indicated. One Step Further Question: What agents should be avoided in patients with atrial fibrillation and Wolf-Parkinson-White (WPW) syndrome? Answer: AV-nodal blocking agents such as adenosine, calcium channel blockers, beta-adrenergic blockers, and digoxin. This can lead to cardiovascular collapse due to preferential accessory pathway conduction.

A 23-year-old man is admitted for a femur fracture. An admission ECG is shown above. What management is indicated? A Atropine B Beta blocker C Reassurance D Transvenous pacing

Correct Answer ( C ) Explanation: The patient's ECG shows a first degree AV block; a normal variant finding in 2% of the population that requires no specific management. First degree AV block is defined as prolonged conduction of atrial impulses. The PR interval > 0.20 seconds (200 msec) and there is no loss of any atrial impulse. All impulses result in a ventricular response. Although it can be a normal variant, it can be seen in pathologic conditions like Lyme disease and should be followed up. However, no specific management is needed. Nodal depressing agents should be avoided or used with caution. Atropine (A) is required for patients with symptomatic bradycardia. Beta blockers (B) are AV nodal blockers and should be used with caution in patients with first degree AV blocks. Transvenous pacing (D) is required for higher degree AV blocks (second degree type II and third degree heart block). One Step Further Question: How does carotid massage affect second degree type I and type II heart block? Reveal Answer: Rapid Review Carotid massage will typically worsen second degree type I block and improve second degree type II block. Heart Block: First Degree ECG will show PR interval > 200 msecs with regular rhythm Treatment is not necessary Comments: most common conduction disturbance and usually represents an incidental finding on an ECG

A 65-year-old man is brought to the ED complaining of nausea for the last two hours. On arrival to the emergency department, he has the cardiac rhythm seen above. His blood pressure is 110/70 mm Hg. He denies any headache, chest pain, or difficulty breathing. Which of the following is the most appropriate next step in management? A Defibrillation B Magnesium sulfate C Procainamide D Synchronized cardioversion

Correct Answer ( C ) Explanation: The rhythm seen in the image is a monomorphic ventricular tachycardia. This occurs in most cases due to the presence of scar tissue in the myocardium. The first objective for treating any patient with a wide-complex tachycardia (WCT) is to evaluate for hemodynamic instability (hypotension, chest pain, dyspnea, altered mental status) because any patient with a WCT can deteriorate quickly as the rhythm degenerates into ventricular fibrillation. In stable patients, *procainamide* is the drug of choice and will terminate the rhythm in the vast majority of cases. It is superior to amiodarone and lidocaine. Not only is it useful in the termination of stable ventricular tachycardia, but it also blocks accessory pathway conduction, which terminates preexcited tachycardias. Sotalol, a class III antidysrhythmic with beta-blocker properties, can also be used in such patients though it is considered a second-line alternative. Defibrillation (A) is reserved for pulseless ventricular tachycardia and ventricular fibrillation. Magnesium sulfate (B) is the treatment of choice for polymorphic ventricular tachycardia (torsades de pointes). If medical management fails or the patient begins to decompensate, the patient should immediately receive synchronized cardioversion (D).

Which of the following is a cause of torsades de pointes? A Drugs that shorten the QT interval B Hyperkalemia C Hypocalcemia D Marijuana use

Correct Answer ( C ) Explanation: Torsades de pointes may be caused by electrolyte disturbances (hypokalemia, hypomagnesemia, hypocalcemia), antiarrhythmic drugs that prolong the QT interval (procainamide, quinidine, disopyramide), N-acetylprocainamide, droperidol, amiodarone, phenothiazines, haloperidol, tricyclic antidepressants, terfenadine, astemizole, ketoconazole, erythromycin, TMP-SMZ, high-dose methadone, or cocaine. Torsades de pointes is also associated with hereditary long QT interval syndromes. It is a form of ventricular tachycardia manifested by episodes of alternating electrical polarity, with the amplitude of the QRS complex twisting around an isoelectric baseline resembling a spindle. The rhythm usually starts with a PVC and is preceded by widening of the QT interval. Treatment involves intravenous infusion of magnesium sulfate and cardioversion. Drugs that prolong the QT interval and not shorten the QT interval (A) can lead to torsades de pointes. Hypokalemia not hyperkalemia (B) leads to torsades de pointes. Cocaine and methadone not marijuana (D) are a cause of torsades de pointes.

A woman with known coronary artery disease presents to the ED with chest pain and ventricular tachycardia. Five minutes after admission she becomes unresponsive. Her rhythm strip is seen above. Which of the following is the most appropriate intervention? A Beta-blockade B Cardioversion C Defibrillation D Vasopressin

Correct Answer ( C ) Explanation: Ventricular fibrillation is the most common dysrhythmia in cardiac arrest patients. Instead of coordinated ventricular depolarization and contraction, ventricular fibrillation (VF) is characterized by rapid disorganized excitation potentials that amount to ineffective contraction. VF occurs with acute infarct or ischemia as well as old infarct or ischemia. Common triggers include electrocution, myocardial ischemia, and hypoxia. Electrocardiographically, VF appears as a chaotic, disorganized waveform that has no discernible typical morphology. Ventricular fibrillation is incompatible with life. Treatment is with immediate defibrillation. Vasopressin (D) was previously part of the ACLS protocol for pulseless ventricular tachycardia (pVT) and ventricular fibrillation (VF). However, the new 2015 AHA guidelines, have removed vasopressin all together. Beta-blockers (A) are not indicated for VF. Cardioversion (B) is only used in patients with organized electrical activity and who have a pulse.

Which of the following best describes the underlying pathophysiology of ventricular tachycardia (NOT supraventricular tachycardia)? A Calcified mitral valve leaflets B Electrical blockade in the atrioventricular node C Irritable ventricular foci D Reentrant pathway exists between atria and ventricles

Correct Answer ( C ) Explanation: Ventricular tachycardia (VT) is usually initiated by irritable ventricular automaticity foci. These foci may become irritable by several factors, three of which are hypokalemia, hypoxia and ischemia. Hypoxia may result from airway obstruction, lack of air (as in suffocation or drowning) and pulmonary compromise (as in pulmonary embolus or pneumothorax). Myocardial ischemia or infarction occurs in disease states such as chronic ischemic heart and coronary disease, or in emergent states, as in hypovolemic shock or cardiogenic shock. The more common causes of irritation are coronary insufficiency (vasospasm as with cocaine, atherosclerosis or thrombosis and embolus) and myocardial infarction. Once a ventricular foci is irritated enough, it will take over as the dominant pacemaker, suppressing the sinoatrial node, and resulting in a tachycardia usually between 150-250 bpm.

After return of spontaneous circulation from a ventricular fibrillation arrest, mortality can be determined by calculating a Cardiac Arrest Score, which takes into account which of the following factors? ALeft ventricular ejection fraction BPre-hospital pharmacologic treatment CSystolic blood pressure DTime to presentation to the emergency department

Correct Answer ( C ) Explanation: Witnessed out-of-hospital cardiac arrest patients can be stratified by using the 3-criteria Cardiac Arrest Score developed by Thompson and McCullough. The criteria are: (1) Systolic blood pressure in the emergency department; (2) time from loss of consciousness to return of spontaneous circulation; (3) neurologic responsiveness. Criteria are SBP > 90 mm Hg (1 point; < 90 mm Hg = 0 points), time from loss of consciousness to return of spontaneous circulation < 25 minutes (1 point; > 25 minutes = 0 points) and positive neurologic responsiveness (1 point; unresponsiveness = 0 points). A score of 0 equates to 90% in-hospital mortality and 3% chance of neurologic recovery. A score of 3 equates to 18% in-hospital mortality and an 89% chance of neurologic recovery. Left ventricular ejection fraction (A), pre-hospital pharmacologic treatment (B) and time to presentation to the emergency department (D) are not criteria in the Cardiac Arrest Score. However, they do play a role in considering the overall prognosis of cardiac arrest. One Step Further Question: What is the greatest risk factor for sudden death from ventricular fibrillation? Answer: Underlying left ventricular dysfunction (LV EF < 30-35%). Rapid Review Cardiopulmonary Arrest Improved outcomes: AED use Early bystander CPR Presenting rhythm: v-tach/v-fib CPR prior to defibrillation Amiodarone use in shock resistant v-tach/v-fib Therapeutic hypothermia

Which of the following states a correct order of electrical current through the heart during one cycle of normal cardiac depolarization? A Atrioventricular node -> sinoatrial node B Bundle of His -> atrioventricular node C Left bundle branch -> right bundle branch D Right bundle branch -> purkinje fibers

Correct Answer ( D ) Explanation: A normal cycle of cardiac depolarization begins in the right atrium's sinoatrial node and passes through the right atrium's internodal tracts to activate the atrioventricular node. The AV node then sends the electrical impulse to the Bundle of His, which then passes current through the left and right bundle branches to the ventricular Purkinje fibers. Atrioventricular (AV) block is characterized as a delay in processing the electrical impulse within the atrioventricular node. This ultimately results in a delay in ventricular depolarization and contraction. There are three main types: first, second and third degree AV block. First-degree AV block is characterized as a prolonged PR interval > 0.2 sec, beginning at the start of the P wave and ending at the start of the QRS complex. Common causes of this type of block include electrolyte abnormalities, enhanced vagal tone (as in athletes), myocarditis or infarction and medications. Common medications which slow cardiac conduction through the AV node and produce a prolonged PR interval include beta and calcium-channel blockers, anticholinesterases and digitalis.

Which of the following treatments is contraindicated in the treatment of a 5-month-old with supraventricular tachycardia? A Adenosine B Ice bag to face C Propranolol D Verapamil

Correct Answer ( D ) Explanation: Due to poor calcium reserves in the sarcoplasmic reticulum in infants, verapamil (calcium channel blocker) use in infants can cause profound hypotension and cardiovascular collapse. Therefore, it should be avoided in patients younger than 12-months-old. Adenosine (A) is the first-line agent to treat supraventricular tachycardia. Applying ice to the face (B) or other vagal maneuvers can be attempted before administration of adenosine. Propranolol (C) is a nonselective beta-blocker used in supraventricular tachycardia, but it is less effective than adenosine.

A 78-year-old man complains of his "heart -stopping". After reviewing his ECG, you appreciate intermittent, large, wide QRS complexes which are not preceded by a P wave. Otherwise, the rhythm strip appears normal without PR interval, ST segment, P wave or T wave abnormalities. Which of the following is the most likely diagnosis? A Atrial fibrillationYour Answer B Multifocal atrial tachycardia C Non-ST-elevation myocardial infarction D Premature ventricular contraction

Correct Answer ( D ) Explanation: Premature ventricular contractions (PVCs) appear as abnormal QRS complexes and T waves that occur in another underlying rhythm. PVCs have 6 characteristics: (1) Occur earlier than the next expected normal QRS; (2) Wider than a normal QRS; (3) QRS morphology is generally bizarre; (4) Preceding P wave is absent; (5) Deflection of the ST segment and T wave is opposite that of the QRS; (6) Followed by a compensatory pause. In general, PVCs are benign. However, when more than 3 occur simultaneously at a rate greater than 100, it is considered ventricular tachycardia. Atrial fibrillation (A) is diagnosed when disorganized electrical activity is seen in place of normal P waves. Apart from the intermittent oddly shaped QRS complex, the remainder of this patient's ECG shows normal P waves. Multifocal atrial tachycardia (B) is characterized by an irregular rhythm with rate > 100 and 3 or more P waves of variable morphology and PR interval variability. NSTEMI (C) is associated with T wave inversion and ST segment depression.

A 50-year-old woman presents to the Emergency Department following a syncopal episode. On physical exam she is diaphoretic. She is alert and oriented. Her blood pressure is 95/50 mm Hg and respirations are 12 per minute. Her rhythm strip is shown above. What is the most appropriate pharmacologic treatment for this patient's condition? Adenosine Amiodarone Amlodipine Atropine

Correct Answer ( D ) Explanation: Sinus bradycardia is a rhythm that originates from the sinoatrial (SA) node with a rate of < 60 beats per minute. The SA node is the heart's normal pacemaker, and sinus bradycardia may be normal in some patients such as conditioned athletes. In these cases, patients will have a resting heart rate of < 60 per minute, but will not exhibit any other symptoms. Other times, sinus bradycardia may be due to organic heart diseases such as coronary artery disease, cardiomyopathy, or myocarditis, resulting in symptomatic sinus bradycardia. Therefore, treatment largely depends on whether the patient is symptomatic (unstable) or not. An unstable patient is anyone who shows signs of poor perfusion, such as altered mental status, diaphoresis, dizziness, or syncope. A stable patient may be monitored closely. Initial pharmacologic treatment for an unstable patient is atropine. Since this patient presents with both a syncopal episode and diaphoresis, she is unstable and atropine is the appropriate therapy. After an initial dose of atropine, infusions of epinephrine or dopamine may also be considered, and for more emergent cases transcutaneous pacing may be necessary. Adenosine (A) is the treatment for supraventricular tachycardia, not bradycardia, and so is not appropriate for this patient. Amiodarone (B) is used to treat dysrhythmias, such as ventricular tachycardia, not a sinus rhythm. Amlodipine (C) is a calcium channel blocker that helps to control heart rate and blood pressure. One Step Further Question: What is the definitive treatment for sinus bradycardia? Answer: Permanent pacemaker

A 37-year-old man presents to the emergency department with chest pain and shortness of breath. His medical history is significant for uncontrolled type I diabetes and end-stage renal disease on hemodialysis. His last dialysis was four days ago and he missed his appointment this morning. His labs are notable for a fingerstick blood glucose 300 mg/dL, potassium 7.0 mmol/L, magnesium 2.0 mEq/L, and phosphorus 4.0 mmol/L. Which of the following findings is most likely to be seen on this patient's ECG? A Osborn waves B QT interval prolongation C U waves D Widened QRS complex

Correct Answer ( D ) Explanation: Symptomatic hyperkalemia is a life-threatening electrolyte abnormality typically seen in patients with underlying acute or chronic kidney disease. It can also be seen in conditions that cause increased tissue breakdown such as tumor lysis syndrome, rhabdomyolysis, and crush injuries. Muscle weakness and paralysis, cardiac conduction abnormalities and cardiac dysrhythmias are the most serious manifestations of hyperkalemia. Symmetrical peaked T waves with a shortened QT interval and ST-T segment depression are the earliest ECG changes seen in patients with hyperkalemia. Worsening hyperkalemia results in progressive lengthening of the PR interval and QRS duration, disappearance of the P wave, and ultimately widening of the QRS complex into a sinusoidal pattern. Treatment of hyperkalemia includes antagonizing the membrane effects of potassium, driving extracellular potassium into cells, and removing potassium from the body. Osborn waves (A), also known as J-waves, are positive deflections in the junction of the QRS complex and the ST segment. They are most commonly seen in patients with hypothermia. QT interval prolongation (B) is seen in hypokalemia, hypomagnesemia, and hypocalcemia. QT interval shortening is seen in hyperkalemia. U waves (C) are small deflections that follow the T wave and are typically seen in hypokalemia, rather than hyperkalemia.

A 39-year-old man presents to the ED complaining of general weakness. He has signs of an upper respiratory infection on exam. His rhythm strip is seen above. Which of the following is the most appropriate next step in management? A Administer 325 mg aspirin and send for a troponin B Apply transcutaneous pacemakers and admit C Consult cardiology D Symptomatic care and discharge

Correct Answer ( D ) Explanation: The ECG denotes first degree AV heart block. This is due to prolonged conduction of atrial impulses without the loss of any single impulse. On the ECG, the PR interval will be >200 ms. This type of block is often a normal variant without clinical significance, occurring in close to 2% of healthy young adults and does not require specific treatment. Although the block is benign, in some patients it may be associated with beta-adrenergic blocker therapy, endocarditis, myocarditis, inferior wall myocardial infarction, hyperkalemia, and hypothermia. Treatment for the patient in this case is supportive, and he can be sent home.

A 55-year-old man presents to the emergency department complaining of palpitations, nausea, and dizziness for 30 minutes. He is afebrile, his pulse is 140 beats per minute, and his blood pressure is 78/50 mm Hg. As the nurse attaches him to the cardiac monitor, you see the rhythm strip seen above. What is the most appropriate treatment? A Adenosine 6 mg IV push B Defibrillation at 360 joules C Procainamide 18 mg/kg infused over 30 minutes D Synchronized cardioversion at 200 joules

Correct Answer ( D ) Explanation: The monitor shows a wide complex tachycardia. Clinically, he is hypotensive and unstable, which necessitates the use of electrical rather than pharmacologic treatment. Synchronized cardioversion is indicated for the treatment of unstable tachydysrhythmias, including certain supraventricular dysrhythmias as well as monomorphic ventricular tachycardia. For treatment of ventricular fibrillation or polymorphic ventricular tachycardia, defibrillation (not synchronized cardioversion) is indicated. Adenosine (A) is a short-acting AV-nodal blocker used frequently in supraventricular dysrhythmias. Its use is not recommended in hypotensive patients with a wide-complex tachycardia. Defibrillating a patient who has a pulse (B) is dangerous and can result in the R-on-T phenomenon. This occurs when a depolarizing impulse (endogenous or exogenous) is delivered during ventricular repolarization (T wave). This can result in polymorphic ventricular tachycardia or ventricular fibrillation. To avoid this complication, synchronized cardioversion coordinates delivery of an electrical impulse so that it occurs with initiation of ventricular contraction (i.e., at the beginning of the QRS complex), thereby avoiding a shock during the relative refractory period of the cardiac cycle. Procainamide (C) is an option for patients with stable ventricular tachycardia, but it is not appropriate treatment of a hypotensive patient with an undifferentiated wide complex tachycardia. Procainamide may also worsen hypotension.

An 18-year-old male undergoes an electrocardiogram as part of a sports physical for his college soccer team. The electrocardiogram was read as abnormal, and he was sent to the ED for evaluation. His electrocardiogram is shown above. Which of the following is the most common complication found with the suspected diagnosis? A Atrial fibrillation B Eisenmenger's syndrome C Type 2 second-degree heart block D Ventricular fibrillationr

Correct Answer ( D ) Explanation: The patient's ECG is suggestive of Brugada syndrome. Brugada syndrome is a hereditary condition that leads to abnormal myocardial depolarization and predisposes otherwise young healthy individuals to ventricular fibrillation and sudden cardiac death. Several variants of Brugada syndrome exist, which involve mutations in the genes responsible for transmembrane sodium, calcium, and potassium ion channels in the heart. The incidence of Brugada syndrome is highest in Asian populations and is lower in groups originating from Western Europe. Most patients with Brugada syndrome are asymptomatic, and the disorder is identified incidentally via an abnormal ECG. Palpitations, near-syncope, and syncope may be reported. There may also be a family history of sudden cardiac death. Classic ECG findings of Type 1 Brugada syndrome include coved ST segment elevation followed by an inverted T wave in leads V1 to V3. If Brugada syndrome is suspected based on ECG findings or clinical history, patients should be referred to cardiology for further testing to rule out structural heart problems and confirm the diagnosis of Brugada syndrome. Medications with sodium channel blockade, such as flecainide and procainamide, should be avoided. The definitive treatment for Brugada syndrome is placement of an implantable cardioverter defibrillator device. Atrial fibrillation (A) occurs in 10 - 20 % of patients with Brugada syndrome and is associated with higher risk for ventricular tachyarrhythmias and disease severity. Eisenmenger's syndrome (B) occurs when a left-to-right cardiac shunt, such as from a ventricular septal defect, switches direction to become a right-to-left cardiac shunt. Type 2 second-degree heart block (C) is not associated with Brugada syndrome.

What is the treatment of choice for the bradycardic component of sick sinus syndrome? A Ablation of accessory pathways B Chronotropic medications C No treatment is necessary D PacemakeR

Correct Answer ( D ) Explanation: Treatment generally requires a permanent pacemaker to prevent sinus arrest. The term sick sinus syndrome was coined to describe patients with SA node dysfunction that causes marked sinus bradycardia or sinus arrest. Often, junctional escape rhythms occur, which may lead to symptoms of lightheadedness and syncope. In some patients with sick sinus syndrome, bradycardic episodes are interspersed with paroxysms of supraventricular tachycardia (atrial fibrillation, atrial flutter, PSVT). Sometimes the bradycardia occurs immediately after spontaneous termination of the tachycardia. An important subset of patients are those with paroxysmal atrial fibrillation that have marked sinus bradycardia and even sinus arrest after spontaneous conversion of atrial fibrillation. The term brady-tachy syndrome has been used to describe patients with sick sinus syndrome who have both tachydysrhythmias and bradydysrhythmias. The diagnosis of sick sinus syndrome and, in particular, the brady-tachy variant often requires monitoring the patient's heart rhythm over several hours, days, or even weeks. A single ECG strip may be normal or may reveal only the bradycardic or tachycardic episode. Ablation of accessory pathways (A) and medications (B) are often used after pacemaker placement to control the tachcycardias. Without pacemaker placement (C), these patients can have sinus arrest. One Step Further Question: What is the most common reason sick sinus syndrome occurs? Answer: Age and replacement of SA node with fibrous tissue. Rapid Review Sick Sinus Syndrome (SSS) SA node dysfunction Tachycardia-bradycardia syndrome: sinus rate varies from fast to slow and back again Syncope, palpitations Definitive rx: pacemaker placement + rate control medication Untreated SSS → sinus block or sinus arrest

A 60-year-old man presents with the ECG seen above. His blood pressure is 80/60 mm Hg. Which of the following is the most likely diagnosis? A Atrial flutter B Atrioventricular reentry tachycardia C Supraventricular tachycardia D Ventricular tachycardia

Correct Answer ( D ) Explanation: Ventricular tachycardia (VT) is present when there are >3 consecutive premature ventricular contractions occurring at a rate > 100. P waves are usually absent and the QRS complexes are wide (> 120 msec) and may be bizarre appearing. Ventricular tachycardia is classified as "monomorphic" (QRS complexes look the same) or "polymorphic" (QRS complexes have varying morphology). Sometimes it is difficult to distinguish ventricular tachycardia from supraventricular tachycardia (SVT) with aberrancy (presence of a bundle branch block). Nonetheless, any patient with a wide complex tachycardia who is hemodynamically unstable should undergo immediate electrical cardioversion. Atrioventricular reentry tachycardia (A) is a type of SVT. It is more commonly associated with regular pulse and monomorphic narrow QRS complexes. Atrial flutter (B) is more commonly associated with regular tachycardia and sawtooth appearance of P waves. It is sometimes difficult to distinguish ventricular tachycardia from supraventricular tachycardia with aberrancy. Any patient with a wide complex tachycardia who is hemodynamically unstable should be treated as having ventricular tachycardia. Some electrocardiographic characteristics that support VT over SVT (C) are extreme left axis deviation, QRS concordance (all QRS complexes from V1 to V6 are either all positive or all negative), and fusion or capture beats, which indicates AV dissociation. One Step Further Question: What is the treatment of hemodynamically unstable ventricular tachycardia? Answer: Electrical cardioversion.

Capture beats and fusion beats confirm the diagnosis of which cardiac dysrhythmia? A Atrial fibrillation B Supraventricular tachycardia C Ventricular fibrillation D Ventricular tachycardia

Correct Answer ( D ) Explanation: Ventricular tachycardia can be diagnosed when capture beats or fusion beats are seen in the setting of a wide complex tachycardia. This is especially helpful when trying to distinguish between ventricular tachycardia and supraventricular tachycardia with aberrant conduction. Supraventricular tachycardia with aberrancy can produce a wide complex tachycardia that mimics ventricular tachycardia. It is important to distinguish between the two because ventricular tachycardia is a more serious condition and treatment is different. During ventricular tachycardia, there is AV dissociation in which independent pacing of the atria and ventricles occurs. The SA node still paces the atria, but the larger ventricular complexes often hide the P waves. However, P waves can be seen occasionally. When a sinus paced depolarization is able to conduct to the ventricles, it can produce a normal-appearing QRS; this is called a capture beat. More commonly, an atrial depolarization is able to conduct only partially before it encounters depolarization coming from the ventricles; this produces a fusion beat, which is a blending of a normal QRS with a PVC like complex. Capture beats and fusion beats do not occur during supraventricular tachycardia, making their presence useful in differentiating the two. Capture and fusion beats are not seen in atrial fibrillation (A), supraventricular tachycardia (B), or ventricular fibrillation (C).

An 81-year-old man with 10 years of coronary artery disease presents with chest pain and altered mental status. His ECG shows consecutive, large and wide QRS complexes. P waves cannot be appreciated. His pulse is 188 bpm. Which of the following is the most likely diagnosis? A Atrial fibrillation B Atrial flutter C Ventricular fibrillation D Ventricular tachycardia

Correct Answer ( D ) Explanation: Ventricular tachycardia is described electrocardiographically as "continuous PVCs", or more specifically, consecutive, fast, large and wide QRS complexes. A wide QRS complex clues one in that the underlying electrical problem is in the ventricles. Ventricular tachycardia can be further defined as monomorphic (QRSs are the same form) versus polymorphic (differing appearance of the QRSs), or sustained (>30 seconds) versus non-sustained (<30 seconds, self-terminates). In this tachydysrhythmia, the ventricles depolarize and contract so fast that cardiac output decreases, especially in the elderly or those with diseased myocardium or coronaries. This perpetuates ventricular ischemia leading to a precipitous decline if the dysrhythmia is not treated. Atrial fibrillation (A) and atrial flutter (B) both have narrow QRS complexes, either irregular or regular respectively. Ventricular fibrilation (C) is identified by a totally erratic appearance of unidentifiable waves. Discrete QRS complexes are absent.

Which of the following diseases is characterized by the presence of a delta wave on an electrocardiogram? Charcot-Marie-Tooth disease Parkinsonism Parkinson's disease Wolff-Parkinson-White syndrome

Correct Answer ( D ) Explanation: Wolff-Parkinson-White (WPW) syndrome is a congenital cardiac condition in which there is an abnormal conduction pathway (accessory bundle) between the atria and ventricles other than the atrioventricular node. This pathway provides the basis for a reentrant circuit which bypasses the typical delay thru the AV node, typically resulting in premature ventricular depolarization and supraventricular tachyarrhythmia. The premature depolarization of some of the ventricle produces a "slurring" of the normal depolarization wave thru the AV node. This is seen as a gradual upsloping of the QR segment, called a delta wave, which creates an illusion of PR interval shortening and QRS lengthening. Charcot-Marie-Tooth disease (A) is a hereditary sensorimotor neuropathy which commonly leads to peroneal muscular atrophy. Parkinson's disease (C) is a degenerative disorder of the dopamine-generating cells in the substantia nigra, which causes parkinsonism symptoms. Parkinsonism (B) is a term used to describe the typical parkinsonian symptoms which arise from a variety of diseases, with Parkinson's Disease being one of these causes. Parkinsonism generally refers to tremor, rigidity, slowed movements and balance impairment. One Step Further Question: What is the name of the accessory bundle in WPW syndrome? Answer: The bundle of Kent. Rapid Review Wolff-Parkinson-White (WPW) Syndrome ECG will show short PR interval, delta wave, wide QRS Most commonly caused by an accessory pathway (bundle of Kent) connects atria to ventricles, bypassing AV node Definitive treatment is radiofrequency ablation

A 35-year-old man presents to the emergency department after a syncopal episode. He describes the sensation of palpitations prior to losing consciousness. Witnesses state he was unconscious for approximately 30 seconds and was not confused upon awakening. ECG in the emergency department shows the rhythm above. Which of the following is the most common dysrhythmia associated with this condition? AAntidromic atrioventricular reentrant tachycardia BAtrial fibrillation CAtrioventricular nodal reentrant tachycardia DOrthodromic atrioventricular reentrant tachycardia

Correct Answer ( D ) Explanation: Wolff-Parkinson-White (WPW) syndrome is a ventricular preexcitation syndrome that involves an anatomic accessory pathway that bypasses the AV node and forms a direct electrical connection between the atria and ventricles. Patients with this accessory pathway are at high risk of supraventricular tachydysrhythmias. The classic triad of WPW syndrome can be seen when the patient is in sinus rhythm: (1) a shortened PR interval (2) a fused QRS complex and (3) a delta wave at the beginning of the QRS complex. These classic signs are typically not seen when the patient is in a tachydysrhythmia. The most common tachydysrhythmia seen in WPW syndrome is atrioventricular reentrant (or reciprocating) tachycardia (AVRT), which can either be antidromic or orthodromic. Orthodromic AVRT is the most common tachydysrhythmia seen in WPW syndrome and accounts for 65% of dysrhythmias seen in WPW patients. It occurs due to anterograde conduction of the atrial impulse through the atrioventricular node with return of the impulse back through the accessory pathway, resulting in an electrical loop. Orthodromic AVRT appears as a narrow-complex tachydysrhythmia and is difficult to distinguish from a typical supraventricular tachycardia. Tachydysrhythmias in WPW should be treated according to the patient's clinical stability, with hemodynamically unstable patients receiving immediate electrical cardioversion. The first intervention in orthodromic AVRT in a hemodynamically stable patient is a vagal maneuver. If this fails, intravenous adenosine can be used, followed by AV blocking agents such as beta blockers or calcium channel blockers. Definitive treatment of WPW syndrome is ablation of the accessory pathway. Antidromic AVRT (A) occurs in 10% of WPW tachydysrhythmias and occurs due to anterograde conduction of the atrial impulse through the accessory pathway and return of the impulse through the atrioventricular node. This results in a wide complex tachycardia. Procainamide is the treatment of choice in antidromic AVRT. Agents that block conduction through the AV node, such as beta-blockers, calcium channel blockers and adenosine, should be avoided as they increase conduction through the aberrant pathway, increasing the risk of ventricular tachydysrhythmias. Atrioventricular nodal reentrant tachycardia (AVNRT) (B) is the most common cause of palpitations in patients without structural heart abnormalities and results from a reentrant electrical circuit in or around the atrioventricular node. The resulting narrow-complex tachydysrhythmia resembles the orthodromic AVRT commonly seen in WPW. Atrial fibrillation (C) is seen in approximately 25% of WPW tachydysrhythmias. Atrial fibrillation in WPW has a very rapid ventricular rate due to the ability of the accessory pathway to conduct the impulse faster than the atrioventricular node and is often a wide-complex tachycardia due to impulses conducted through both pathways fusing to form a depolarization. One Step Further Question: What class antiarrhythmic is procainamide? Answer: Vaughn Williams Class 1a. Rapid Review Wolff-Parkinson-White (WPW) Syndrome ECG will show short PR interval, delta wave, wide QRS Most commonly caused by an accessory pathway (bundle of Kent) connects atria to ventricles, bypassing AV node Definitive treatment is radiofrequency ablation

Which of the following is associated with a shortened PR interval? A Brugada Syndrome B Mobitz 2° AV Block C Wellens Syndrome D Wolff-Parkinson-White syndrome

Correct Answer ( D ) Explanation: Wolff-Parkinson-White syndrome (WPW) is a congenital condition in which there is an abnormal accessory conduction pathway called the bundle of Kent. This accessory conduction pathway causes a premature depolarization of the ventricles by bypassing the AV node. The AV node typically acts to delay depolarization (the PR interval) to the ventricles to allow them to fill before contracting (QRS complex). The accessory pathway causes early depolarization of the ventricles resulting in an early QRS complex, thereby shortening the PR interval. On an ECG, this is represented by an upslope of the QRS complex (delta wave) that causes a widened QRS complex (>120 msec). The primary significance of WPW syndrome is that it predisposes patients to the development of tachydysrhythmias, particularly atrial fibrillation. Brugada Syndrome (A) is a hereditary condition characterized by a right bundle branch block-like pattern with ST elevation in Leads V1 to V3. Wellens Syndrome (C) is described by a large inverted T wave in Leads V2 and V3 and is associated with critical stenosis of the proximal left anterior descending coronary artery. Both Brugada and Wellens syndromes have normal PR intervals. Mobitz 2° AV Block (B) occurs at the level of AV node. Depolarizations from the atria are blocked and do not reach the ventricles, resulting in some P waves not being followed by a QRS complex (thereby, having no PR interval). The P waves that are followed by QRS complexes have normal PR intervals.

A 72-year-old woman presents to the ED with an acute onset of dyspnea and palpitations that began four hours prior to arrival. Vital signs include a heart rate of 144 beats per minute, blood pressure of 80/50 mm Hg, respiratory rate of 28 breaths per minute, temperature of 37.0°C, and a pulse oximetry of 88% on room air. The above 12-lead ECG is obtained. What is the most appropriate next step in the management of this patient? A Anticoagulation with enoxaparin followed by warfarin B Chemical cardioversion using procainamide C Rate control using diltiazem D Rate control using esmolol E Synchronized cardioversion

Correct Answer ( E ) Explanation: This patient has atrial fibrillation with a rapid ventricular rate and is hemodynamically unstable (BP 80/50 mm Hg and pulse ox 88%). This dysrhythmia needs to be emergently corrected in order to stabilize the patient; failure to do so could result in sudden cardiac death. In such circumstances, emergent synchronized cardioversion is required. If the patient is hemodynamically stable, the approach to management (rate vs. rhythm control) depends on the time of dysrhythmia onset. If the onset is unknown or greater than 48 hours, then cardioversion should be delayed until the patient can be adequately anticoagulated with enoxaparin (A) followed by warfarin. In the interim, rate control would be accomplished with either esmolol (D), a short acting ß-blocker, or diltiazem (C), a calcium channel blocker. For patients with stable paroxysmal atrial fibrillation and duration of onset < 48 hours, chemical cardioversion using procainamide (B) can be attempted. In unstable patients, regardless of the rhythm duration, synchronized electrical cardioversion is recommended. Alternatives to procainamide include amiodarone, ibutilide, and, to a lesser degree, flecainide.

Rapid Review Wolff-Parkinson-White (WPW) Syndrome

ECG will show short PR interval, delta wave, wide QRS Most commonly caused by an accessory pathway (bundle of Kent) connects atria to ventricles, bypassing AV node Definitive treatment is radiofrequency ablation

Rapid Review Sick Sinus Syndrome (SSS)

SA node dysfunction Tachycardia-bradycardia syndrome: sinus rate varies from fast to slow and back again Syncope, palpitations Definitive rx: pacemaker placement + rate control medication Untreated SSS → sinus block or sinus arrest

Which of the following drugs is most likely to be associated with the development of atrial tachydysrhythmias? A Ethanol B Gamma hydroxybutyrate (GHB) C Lorazepam D Phenobarbital

Your Answer Correct Answer ( A ) Explanation: Ethanol abuse is associated with the development of atrial dysrhythmias, specifically, atrial fibrillation. Alcohol ingestion (acute or chronic) has multiple effects on the cardiovascular system. It can exacerbate coronary artery disease, lead to cardiomyopathy and produce dysrhythmias. Left ventricular dysfunction is common in patients with moderate alcohol consumption. Additionally, these patients may have diastolic dysfunction. Supraventricular and ventricular dysrhythmias are common. The so called "holiday heart" that occurs with heavy drinking can present as atrial fibrillation or, in unusual cases, ventricular tachycardia. Additionally, electrolyte deficiencies (hypokalemia and hypomagnesemia) predispose to dysrhythmias. GHB (B) has been associated with bradycardia and hypotension. Lorazepam (C) and phenobarbital (D) are generally not associated with cardiac dysrhythmias.

Which one of the following cardiac dysrhythmias is associated with antipsychotic use? A Atrial fibrillation B Third degree heart block C Torsades de pointes D Wolff-Parkinson-White syndrome

Your Answer Correct Answer ( C ) Explanation: All antipsychotic agents can prolong ventricular repolarization, leading to a prolonged QT interval, which can in turn lead to torsades de pointes and sudden cardiac death. Although all antipsychotics can affect ECG intervals, the agents with the greatest propensity to prolong QTc are thioridazine, pimozide, droperidol, and ziprasidone. The incidence of sudden cardiac death among patients taking antipsychotics is about twice that of the general population. Physicians should avoid combining antipsychotic medications with other medications that prolong the corrected QT interval such as classes I and III antidysrhythmics, tricyclic antidepressants, and certain antibiotics. Wolff- Parkinson-White syndrome (D) is characterized by a shortened PR interval, slurring and slow rise of the initial upstroke of the QRS complex known as the delta wave and widened QRS complex on ECG. It has a large genetic component and is not caused by medications such as antipsychotics. Atrial fibrillation (A) has strong associations with many cardiovascular diseases as well as hyperthyroidism, catecholamine excess, hemodynamic stress, metabolic stress, and other causes however antipsychotic medications are not known to cause atrial fibrillation. Third degree heart block (B) can develop from isolated single-agent overdose or from combined or iatrogenic coadministration of digoxin, beta-adrenergic, calcium channel blocking agents and other antiarrhythmics.

A 50-year-old woman with a history of mitral stenosis secondary to rheumatic fever presents with atrial fibrillation. She does not have a history of heart failure, hypertension, diabetes mellitus or previous stroke or transient ischemic attack. What is the most appropriate management for this patient?

warafin


Conjuntos de estudio relacionados

Missouri Compromise, Compromise of 1850 and The Fugitive Slave Law

View Set

Fundamentals of Nursing - Physiological Aspects of Care

View Set

Adolescence & Emerging Adulthood: Ch. 9, 10, 11,

View Set

NPB110A Quizzes for MT2 (Quiz 3 and 4)

View Set